Nothing Special   »   [go: up one dir, main page]

TEST-7: Vision

Download as pdf or txt
Download as pdf or txt
You are on page 1of 60

English Medium Test -7

VISIONIAS
www.visionias.in

TEST-7
Test Booklet Series

GENERAL STUDIES (P) 2023 – Test – 3791


C
Time Allowed: Two Hours Maximum Marks: 200

India Economy
National Income Accounting Capital Market
▪ Definition ▪ Terms and Concepts
▪ Concepts: GDP, GNP, Agriculture
▪ Measurement Industry
Money and Banking
Services
▪ Function of Money, Evolution of Money
Infrastructure and Communication
etc.
▪ Banking: Need of Banks, Functions, Types, Five year Plans
Banking Reforms, Central Bank and its Economic Reforms since 1991
role Foreign Trade
Inflation and Employment India and IMF, WTO, WIPO
▪ Definition, types etc. India and International Institutions
Government Budgeting and Fiscal Policy IPRs
▪ Budgeting, types, its evolution in India,
Process, Component etc. + Current Affairs (May 2022)
▪ Public finance, Revenue and Expenditure,
Deficits, Taxes, Debts etc.
External Sector and Currency Exchange
rates
▪ BoP, BoT, International trade, IMF, WTO,
Trade policies, issues and challenges etc.
2023
www.visionias.in
1
DO NOT OPEN THIS BOOKLET UNTIL YOU ARE ASKED TO DO SO
©Vision IAS
VISIONIAS
www.visionias.in

Test Booklet Series

TEST BOOKLET

GENERAL STUDIES (P) 2023 – Test – 3791


C
Time Allowed: Two Hours Maximum Marks: 200

INSTRUCTIONS

1. IMMEDIATELY AFTER THE COMMENCEMENT OF THE EXAMINATION, YOU SHOULD CHECK THAT THIS BOOKLET
DOES NOT HAVE ANY UNPRINTED OR TURN OR MISSING PAGES OR ITEMS, ETC. IF SO, GET IT REPLACED BY A
COMPLETE TEST BOOKLET.

2. ENCODE CLEARLY THE TEST BOOKLET SERIES A, B, C OR D AS THE CASE MAY BE IN THE APPROPRIATE PLACE IN
THE ANSWER SHEET.

3. You have to enter your Roll Number on the Test Booklet in the Box
provided alongside. Do NOT write anything else on the Test Booklet.

4. This Test Booklet contains 100 items (Questions). Each item is printed in English. Each item comprises four
responses (answers). You will select the response which you want to mark on the Answer Sheet. In case you
feel that there is more than one correct response with you consider the best. In any case, choose ONLY ONE
response for each item.

5. You have to mark all your responses ONLY on the separate Answer Sheet provided. See direction in the
answers sheet.

6. All items carry equal marks. Attempt all items. Your total marks will depend only on the number of correct
responses marked by you in the answer sheet. For every incorrect response 1/3rdof the allotted marks will be
deducted.

7. Before you proceed to mark in the Answer sheet the response to various items in the Test booklet, you have to
fill in some particulars in the answer sheets as per instruction sent to you with your Admission Certificate.

8. After you have completed filling in all responses on the answer sheet and the examination has concluded, you
should hand over to Invigilator only the answer sheet. You are permitted to take away with you the Test
Booklet.

9. Sheet for rough work are appended in the Test Booklet at the end.

DO NOT OPEN THIS BOOKLET UNTIL YOU ARE ASKED TO DO SO


1 www.visionias.in ©Vision IAS
1. With reference to Market Intervention 3. With reference to the Marginal Standing

Scheme (MIS), consider the following Facility (MSF) and Statutory Liquidity Ratio

statements: (SLR), consider the following statements:


1. The objective of this scheme is to 1. MSF refers to the rate at which the
promote the domestic production of scheduled banks can borrow funds
those food crops that India imports from overnight from RBI against government
abroad. securities.
2. Under the MIS, a pre-determined 2. SLR is a tool for controlling liquidity in
quantity at a fixed Market Intervention the domestic market via manipulating
Price (MIP) is procured by NAFED as bank credit.
the central agency. 3. MSF is always fixed above the repo
Which of the statements given above is/are rate.
correct?
Which of the statements given above is/are
(a) 1 only
correct?
(b) 2 only
(a) 1 and 2 only
(c) Both 1 and 2
(b) 2 and 3 only
(d) Neither 1 nor 2
(c) 3 only

(d) 1, 2, and 3
2. Which of the following forms a part of the

Revenue Expenditure of the Government of


4. The Bretton Woods Agreement led to the
India?
creation of which of the following
1. Grants given to state government for
Institutions?
asset creation
1. World Bank
2. Subsidies and pensions given to widows
2. World Trade Organisation
3. Purchase of Rafale aircraft
3. International Monetary Fund
4. Interest payment on debt incurred by the
4. World Economic Forum
government
Select the correct answer using the code
Select the correct answer using the code
given below.
given below.
(a) 1 and 2 only
(a) 1 and 4 only
(b) 2 and 4 only
(b) 2 and 3 only
(c) 1 and 3 only
(c) 1, 2, 3 and 4
(d) 3 and 4 only
(d) 1, 2 and 4 only
2 www.visionias.in ©Vision IAS
5. Consider the following statements regarding 8. Which of the following forms a part of the
the unorganized sector in India: internal debt of India?
1. In the unorganized sector, the maximum 1. Securities issued to international
number of workers employed in an financial institutions
enterprise is fifty. 2. Dated securities
2. The term ‘unorganized worker’ is not
3. Market Stabilization Scheme bonds
defined in India under any act of
4. NRI deposits
government.
Select the correct answer from the code
Which of the statements given above is/are
given below.
correct?
(a) 1 only (a) 1 and 4 only

(b) 2 only (b) 3 and 4 only


(c) Both 1 and 2 (c) 1, 2 and 3 only
(d) Neither 1 nor 2 (d) 2 and 3 only

6. With reference to Web 5.0, consider the 9. If the Reserve Bank of India (RBI) increases
following statements: the bank rate, what effects it could possibly
1. Web 5.0 envisions an Internet without a have on the Indian economy?
threat of censorship from governments 1. Loan taken by the commercial banks
or big tech.
becomes cheaper.
2. Web 5.0 will allow users to own their
2. It decreases the money supply in the
identity and data.
economy.
Which of the statements given above is/are
Select the correct answer using the code
correct?
(a) 1 only given below.

(b) 2 only (a) 1 only


(c) Both 1 and 2 (b) 2 only
(d) Neither 1 nor 2 (c) Both 1 and 2
(d) Neither 1 nor 2
7. Consider the following statements regarding
Green Box Subsidies under the World Trade 10. Consider the following statements regarding
Organization: Currency Deposit Ratio (CDR):
1. Under WTO, Green Box subsidies can 1. It is the ratio of money held by the
be increased without any financial
public in currency to that they hold in
limitation.
bank deposits.
2. Subsidies provided under India's Public
2. It reflects people’s preference for
Distribution System (PDS) come under
liquidity.
Green Box Subsidies.
Which of the statements given above is/are
Which of the statements given above is/are
correct? correct?
(a) 1 only (a) 1 only
(b) 2 only (b) 2 only
(c) Both 1 and 2 (c) Both 1 and 2
(d) Neither 1 nor 2 (d) Neither 1 nor 2

3 www.visionias.in ©Vision IAS


11. With reference to the Fiscal Responsibility 14. With reference to tribunals and high courts,
and Budget Management(FRBM) Act, consider the following statements:
consider the following statements: 1. While high courts are established by the
1. The FRBM Act bans the purchase of Constitution, tribunals can be established
primary issues of the Central by Acts of Parliament only.
Government securities by the RBI. 2. Like high courts, tribunals consist of
2. The rules under the FRBM Act aim to judicial members only.
eliminate the fiscal deficit of the Central 3. Both tribunals and high courts have the
Government.
power to use all the enacted laws before
Which of the statements given above is/are
making a decision.
correct?
Which of the statements given above is/are
(a) 1 only
correct?
(b) 2 only
(a) 1 only
(c) Both 1 and 2
(b) 2 and 3 only
(d) Neither 1 nor 2
(c) 3 only
12. Which of the following statements is correct (d) None
regarding the IMF's Special Drawing Rights
(SDR)? 15. Consider the following statements regarding
(a) SDR is the official currency of the the local area banks:
International Monetary Fund (IMF). 1. They are set up to enable the
(b) Value of the SDR is based on a basket of mobilization of rural savings by local
four currencies. institutions.
(c) SDRs can be held by private entities or 2. The priority sector lending targets are
individuals. applicable to local area banks.
(d) The value of the SDR is determined Which of the statements given above is/are
daily based on market exchange rates. correct?
(a) 1 only
13. Consider the following statements regarding (b) 2 only
the approaches used to measure (c) Both 1 and 2
unemployment:
(d) Neither 1 nor 2
1. Usual status approach to measure
unemployment uses seven days
16. Consider the following statements with
preceding the date of the survey as the
reference to National e-Vidhan Application
reference period.
(NeVA):
2. Current daily status approach to measure
1. It is a work-flow system deployed on
unemployment uses each day of the
seven days preceding the date of the NIC Cloud, MeghRaj.
survey as the reference period. 2. It is a mission mode project to digitize
3. The usual status approach to measure and make the functioning of State
unemployment fails to capture the short- Legislatures paperless.
term fluctuations in employment. Which of the statements given above are
Which of the statements given above is/are correct?
correct? (a) 1 only
(a) 1 only (b) 2 only
(b) 2 and 3 only (c) Both 1 and 2
(c) 1, 2 and 3 (d) Neither 1 nor 2
(d) 1 and 2 only
4 www.visionias.in ©Vision IAS
17. With reference to India-Gulf relations, 20. India is a member of which of the following
consider the following statements: multilateral export control regimes?
1. Out of India’s total import of crude oil in 1. Wassenaar Arrangement
2. Nuclear Suppliers Group
last 15 years, the share of Gulf countries
3. Australia Group
has been more than 80%. 4. Missile Technology Control Regime
2. In last 5 years, Saudi Arabia has been Select the correct answer using the code
India’s largest trading partner in the given below.
region. (a) 1, 3 and 4 only
Which of the statements given above is/are (b) 2 and 4 only
(c) 1 and 4 only
correct?
(d) 1 and 3 only
(a) 1 only
(b) 2 only 21. While calculating the Gross Domestic
(c) Both 1 and 2 Product (GDP) by expenditure method,
(d) Neither 1 nor 2 which of the following factors are taken into
account?
1. Final household consumption
18. In the context of "Deficit" in the Annual
expenditure
Financial Statement, consider the following 2. Expenditure on intermediate goods
statements: 3. Final capital expenditure
1. Effective Revenue Deficit signifies that 4. Government expenditure on
amount of capital receipts that are being unemployment allowance
used for actual consumption expenditure Select the correct answer using the code
given below.
of the government.
(a) 1 and 4 only
2. Gross Primary Deficit refers to the (b) 1, 3 and 4 only
difference between the Gross fiscal (c) 1 and 3 only
deficit and the Net interest liabilities. (d) 2, 3 and 4 only
Which of the statements given above is/are
correct? 22. With reference to World Bank Group,
consider the following pairs:
(a) 1 only
Institutions Objectives
(b) 2 only
1. International : Reduce poverty by
(c) Both 1 and 2
Finance Corporation providing zero to
(d) Neither 1 nor 2 low-interest loans.
2. International : Encourage growth
19. He sat on a piece of rock for 13 continuous Development of the private sector
days when challenged about the authenticity Association in developing
of the Abhyangas he had written. His countries.
3. Multilateral : Promote foreign
message about a casteless society and his
Investment Guarantee direct investment
denial of rituals had led to a social
Agency into developing
movement. He is credited with starting the countries.
Wari pilgrimage. This is the most likely Which of the pairs given above is/are
description of who among the following? correctly matched?
(a) Chaitanya Mahaprabhu (a) 1 only
(b) Vallabhacharya (b) 2 and 3 only
(c) 1 and 3 only
(c) Tukaram
(d) 3 only
(d) Tulsidas
5 www.visionias.in ©Vision IAS
23. Which of the following statements about the 26. In the context of the Indian economy,
calculation of Gross Domestic Product consider the following statements:
(GDP), is not correct? 1. The unemployment rate is defined as the
(a) The GDP measures the aggregate percentage of unemployed persons in the
production of final goods and services labor force.
taking place within the domestic 2. The worker population ratio is defined
territory. as the percentage of total workers to the
(b) It does not take into account the total population.
aggregate production of final goods and Which of the statements given above is/are
services produced by foreign national
correct?
within the domestic territory.
(a) 1 only
(c) In GDP calculation, negative
(b) 2 only
externalities are not taken into account.
(c) Both 1 and 2
(d) In GDP calculation, the aggregate
(d) Neither 1 nor 2
production of final goods produced by
Indian citizen in the foreign territory is
27. Consider the following statements regarding
not included.
the liquid nano urea:
24. Which of the following statements is/ are not 1. It helps in minimizing the environmental
correct regarding the Green Revolution in footprint.
India? 2. It has about 10,000 times more surface
1. Green Revolution started in India in the area to volume size in comparison to the
early 1950s. conventional granular urea.
2. It was marked by an increase in the use 3. It is indigenously developed and
of High-Yielding Variety (HYV) seeds. patented by Indian Farmers Fertilizer
3. The HYV seeds significantly lowered Cooperative Limited.
the use of irrigation in India Which of the statements given above is/are
Select the correct answer using the code correct?
given below. (a) 1, 2 and 3
(a) 1 and 3 only (b) 2 and 3 only
(b) 2 only (c) 1 and 3 only
(c) 1, 2 and 3 (d) 1 only
(d) 3 only
28. Which of the following can be the potential
25. With reference to the GDP Deflator, traits of a Contractionary Fiscal Policy?
consider the following statements:
1. Decrease in tax rates of vehicles in order
1. It is the ratio of the nominal GDP to the
to increase the sale
real GDP.
2. Reduction in subsidies of LPG cylinders
2. The weights of goods used in calculating
3. Increase in government expenditure on
GDP deflator differ according to their
infrastructure development
production level.
Select the correct answer from the code
Which of the statements given above is/are
given below.
correct?
(a) 1 only (a) 2 only
(b) 2 only (b) 1, 2 and 3
(c) Both 1 and 2 (c) 1 and 3 only
(d) Neither 1 nor 2 (d) 3 only

6 www.visionias.in ©Vision IAS


.
29. With reference to the Cash Reserve Ratio 32. Which of the following can be accounted as
(CRR), consider the following statements: Capital Receipts in the Annual Financial
1. It is a percentage of deposits which a Statement?
commercial bank must keep with itself 1. Loans received from foreign
in reserves. governments
2. Non-Bank Financial Corporations 2. Borrowings by the government through
(NBFCs) are outside the purview of this the sale of Treasury Bills
reserve requirement. 3. Collection and appropriation of direct
Which of the statements given above is/are tax
correct?
4. Recovery of loans and advances
(a) 1 only
Select the correct answer from the code
(b) 2 only
given below.
(c) Both 1 and 2
(a) 2 and 4 only
(d) Neither 1 nor 2
(b) 1 and 3 only
(c) 1, 2 and 4 only
30. A rise in the price of foreign exchange (a fall
(d) 1, 2, 3 and 4
in the value of rupee) would likely result in
which one of the following?
33. Which one of the following best describes
(a) Decrease the cost (in terms of rupees) of
the term 'money multiplier'?
purchasing a foreign good.
(a) It is the demand for money for carrying
(b) Increase the cost (in terms of rupees) of
out transactions.
purchasing a foreign good.
(b) It is the ratio of total money supply to
(c) Decrease in India’s exports with other
the stock of high powered money in an
things remaining constant.
(d) Both (b) and (c) economy.
(c) It is the fraction of the total deposits

31. With reference to Consumer Price Index which commercial banks keep as

(CPI), consider the following statements: reserves.

1. The CPI takes into account the prices of (d) It is the demand for money as a store of

goods only. wealth.


2. It includes the prices of imported goods.
Which of the statements given above is/are 34. Often seen in the news, Strait of Malacca
correct? connects
(a) 1 only (a) Red Sea and Arabian Sea.
(b) 2 only (b) Andaman Sea and South China Sea.
(c) Both 1 and 2 (c) South China Sea and East China Sea.
(d) Neither 1 nor 2 (d) Bay of Bengal and Arabian Sea.

7 www.visionias.in ©Vision IAS


.
35. In the context of minerals in India, which of 37. Which of the following best describes

the following is/are classified as major 'Casualisation of Workforce'?

minerals? (a) It is the process wherein there is a


continuous and gradual shift of the
1. Coal
workforce from informal to formal
2. Uranium
sector.
3. Gold
(b) It is a long-term process that involves
4. Iron ore bringing companies under the regulatory
Select the correct answer using the code regime of government
given below. (c) It refers to the increase in number of

(a) 2, 3 and 4 only self-employed persons as percentage of

(b) 2 and 4 only workforce.


(d) It is the process of moving from self-
(c) 1, 2, 3 and 4
employment and regular salaried
(d) 3 only
employment to casual wage.

36. Rakesh was manual labour in a factory that 38. Which one of the following is used as the
used to manufacture shoes. The area where official classification criteria for Micro,
the factory was located was bought by a Small, and Medium Enterprises (MSME) in

foreign entity that specialized in shoe India?


(a) Number of workers employed
manufacturing. The machines for the
(b) Share in exports of total manufactured
manufacture and design of shoes were
goods
imported from the United States of America
(c) Investment and annual turnover
and manufacturing plants were set up in the
(d) Number of units of electricity consumed
area. It was observed that special knowledge

of computers and Numerical competency 39. Which of the following statements is correct
was required to operate those machines regarding the Indus Water Treaty (IWT)?

which Rakesh lacked and thus he was fired (a) It was signed between India, Pakistan
and China.
from his job leading to unemployment.
(b) Under this, waters of three rivers namely
What kind of unemployment is depicted in
Ravi, Sutlej and Jhelum have been
the passage given above?
allocated exclusively to India.
(a) Structural Unemployment
(c) The World Bank is a guarantor to the
(b) Frictional Unemployment treaty.
(c) Seasonal Unemployment (d) It requires parties to meet at least once in
(d) Disguised Unemployment five years.

8 www.visionias.in ©Vision IAS


40. Foreign Portfolio Investment (FPI) by any 43. The cases of monkeypox recently have been
confirmed, with Europe being the worst
single investor group cannot exceed X% of
affected. In this context, which of the
the equity of an Indian company, beyond following statements is correct about the
which it will be treated as Foreign Direct monkeypox?
(a) A person with monkeypox remains
Investment (FDI). The value of X is infectious for life even after treatment.
(a) 10 (b) Sexual activity is the sole mode of its
transmission.
(b) 20 (c) Anyone can get or pass on monkeypox
(c) 25 including rodents and primates.
(d) It is caused by the same Variola virus
(d) 50
which causes smallpox.

44. Soil Health Cards (SHC) are printed report


41. Consider the following statements regarding
cards issued to farmers once in three years
Balance of Payments (BoP): indicating the status of the soil. Which of the
1. The BoP of a country comprises following are expected consequences of Soil
Health Cards?
transactions between residents and non- 1. Balanced application of fertilizers
residents during a period. 2. Reduced import bill
3. Increased financial strain on the
2. A country having a balance of payments government
equilibrium will experience an increase 4. Better land management
Select the correct answer using the code
in foreign exchange reserves.
given below.
Which of the statements given above is/are (a) 1 and 2 only
(b) 1, 2 and 4 only
correct?
(c) 3 and 4 only
(a) 1 only (d) 1, 2, 3 and 4
(b) 2 only
45. With reference to the Hybrid Annuity Model
(c) Both 1 and 2 (HAM) model for Public-Private Partnership
(d) Neither 1 nor 2 in highway construction, consider the
following statements:
1. As per the model, the government will
42. In the context of poverty categorization in contribute 60% of the project cost in the
first five years through annual payments
India, which of the following best describes
(annuity).
'churning poor'? 2. Under HAM, a road developer
constructs the road and he/she is allowed
(a) People who are always poor.
to recover his/her investment through
(b) People with high annual income but low toll collection.
standard of living. Which of the statements given above is/are
correct?
(c) People who regularly move in and out of (a) 1 only
poverty. (b) 2 only
(c) Both 1 and 2
(d) Wealthy people with low literacy. (d) Neither 1 nor 2

9 www.visionias.in ©Vision IAS


46. Consider the following statements regarding 48. Consider the following statements regarding
patent rights in India: the Neobanks:
1. They are the fintech firms, with only a
1. Patent is a statutory right for an
digital presence.
invention granted to the patentee by the 2. They do not have a bank license of their
Government. own.
2. Patent is granted for 50 years from the 3. They offer debit cards.
Which of the statements given above is/are
date of filing of an application.
correct?
3. An invention relating to a process cannot (a) 1 and 2 only
be patented in India. (b) 2 only
(c) 1 and 3 only
Which of the statements given above is/are
(d) 1, 2 and 3
correct?
(a) 1 only 49. With reference to the different systems of
(b) 2 and 3 only taxation, consider the following statements:
1. The percentage tax rate increases
(c) 1 and 3 only
proportionally with the increase in the
(d) 1 and 2 only income under the Proportional Taxation
Regime.
47. Consider the following statements with 2. A proportional income tax makes
disposable income as well as consumer
reference to voting in the Rajya Sabha
spending more sensitive to fluctuations
elections: in GDP.
1. Open ballot voting is done in the Which of the statements given above is/are
correct?
elections to the Council of States.
(a) 1 only
2. Members of the Legislative Assembly
(b) 2 only
(MLAs) belonging to a political party (c) Both 1 and 2
can show their ballot papers after (d) Neither 1 nor 2

marking their vote to the authorised


50. Consider the following statements regarding
agent of that party only. a socialist economy:
3. Independent MLAs can show their 1. In a socialist economy, the government
marked ballot paper to the authorised decides what goods are to be produced .
2. Distribution of resources under
agent of any party.
socialism is supposed to be based on the
Which of the statements given above are purchasing power of the people.
correct? Which of the statements given above is/are
(a) 1 and 2 only correct?
(a) 1 only
(b) 2 and 3 only
(b) 2 only
(c) 1 and 3 only (c) Both 1 and 2
(d) 1, 2 and 3 (d) Neither 1 nor 2

10 www.visionias.in ©Vision IAS


51. Consider the following statements with 54. Which of the following can be regarded as
reference to Leadership for Industry the quantitative tools of the Reserve Bank of
Transition (LeadIT): India, to control money supply in the
1. It is a voluntary initiative launched by economy?
India and the United Kingdom at the UN 1. Changing the cash reserve ratio
Climate Action Summit in 2020.
2. Open Market Operations
2. Its Secretariat is hosted by the World
3. Moral suasion
Economic Forum.
4. Changing the Bank Rate
Which of the statements given above is/are
Select the correct answer using the code
correct?
(a) 1 only given below.
(b) 2 only (a) 1 and 4 only
(c) Both 1 and 2 (b) 2 and 3 only
(d) Neither 1 nor 2 (c) 1, 2 and 4 only
(d) 1, 2, 3 and 4
52. Consider the following statements regarding
the Indian Agriculture sector since the 1991 55. Consider the following statements regarding
reforms: the Flexible Exchange Rate System:
1. Percentage share of public investment in
1. In this system, the exchange rate is
the agriculture sector has increased
determined by the market forces of
steadily.
demand and supply.
2. There has been a shift from production
2. At present, India follows a completely
for the domestic market towards
production for the export market. flexible exchange rate system.
Which of the statements given above Which of the statements given above is/are
is/are correct? correct?
(a) 1 only (a) 1 only
(b) 2 only (b) 2 only
(c) Both 1 and 2 (c) Both 1 and 2
(d) Neither 1 nor 2 (d) Neither 1 nor 2

53. With reference to monetized deficit as a 56. Consider the following statements regarding
method to finance debt, consider the
the Agreement on Trade-Related Aspects of
following statements:
Intellectual Property Rights (TRIPS):
1. Monetized deficit is that part of the
1. The TRIPS agreement applies to all
Government deficit which is financed
solely by borrowing from the Reserve WTO members.
Bank of India. 2. The agreement allows compulsory
2. Monetized deficit includes short-term licensing and government use of a patent
finance instruments as well as long-term without the authorization of its owner.
securities. Which of the statements given above is/are
Which of the statements given above is/are correct?
correct? (a) 1 only
(a) 1 only (b) 2 only
(b) 2 only (c) Both 1 and 2
(c) Both 1 and 2
(d) Neither 1 nor 2
(d) Neither 1 nor 2

11 www.visionias.in ©Vision IAS


57. Consider the following statements with 60. With reference to Inflation-Indexed Bonds,
reference to the convertibility of currencies: Consider the following statements:
1. Convertibility refers to the ability to 1. The Inflation-Indexed Bonds provide
convert domestic currency into foreign protection only to principal and not to
currencies and vice versa. interest payments.
2. Currently, India has partial convertibility 2. Inflation-Indexed Bonds are treated as
for both current and capital accounts. G-Secs and are thus eligible to be kept
Which of the statements given above is/are as part of the Statutory Liquidity Ratio
correct? requirements of banks.
(a) 1 only Which of the statements given above is/are
(b) 2 only correct?
(c) Both 1 and 2 (a) 1 only
(d) Neither 1 nor 2 (b) 2 only
(c) Both 1 and 2
58. Which of the following are functions of the (d) Neither 1 nor 2
Central Bank (RBI) in the Indian economy?
1. It issues the currency of the country. 61. With reference to the National Disposable
2. It controls the money supply of the Income, consider the following statements:
country. 1. It comprises Net National Product at
3. It is the custodian of the foreign market price and net other current
exchange reserves of the economy. transfers from the rest of the world.
4. It acts as a bank to the banking system in 2. It gives an idea about the maximum
the economy. amount of goods and services that the
Select the correct answer using the code domestic economy has at its disposal.
given below. Which of the statements given above is/are
(a) 1, 2 and 3 only correct?
(b) 1, 3 and 4 only (a) 1 only
(c) 2 and 4 only (b) 2 only
(d) 1, 2, 3 and 4 (c) Both 1 and 2
(d) Neither 1 nor 2
59. Consider the following statements regarding
the open market operations done by the 62. Consider the following statements in the
Reserve Bank of India (RBI): context of public goods:
1. It refers to buying and selling of the 1. Consumption of public goods by an
bonds issued by the Government in the individual does not reduce the amount
open market. available for consumption of the same
2. Purchase of government securities from good for another individual.
the bond market by the RBI, increases 2. Public Provision goods are those goods
the high powered money in the that are produced directly by the
economy. government.
Which of the statements given above is/are Which of the statements given above is/are
correct? correct?
(a) 1 only (a) 1 only
(b) 2 only (b) 2 only
(c) Both 1 and 2 (c) Both 1 and 2
(d) Neither 1 nor 2 (d) Neither 1 nor 2

12 www.visionias.in ©Vision IAS


63. Which of the following is not correct 66. Consider the following statements:
regarding human capital? 1. Until the outbreak of the First World
War, all currencies were pegged to the
(a) Human capital is intangible and cannot
value of gold.
be easily sold in the market. 2. Post-independence the Indian rupee was
(b) Unlike physical capital, human capital pegged to the pound sterling till 1975.
Which of the statements given above is/are
does not depreciate with time.
correct?
(c) Human capital benefits not only the (a) 1 only
owner but also the society in general. (b) 2 only
(c) Both 1 and 2
(d) Human capital is not perfectly mobile
(d) Neither 1 nor 2
between countries.
67. Consider the following statements regarding
64. Which one of the following is not a the Small Finance Banks in India:
1. They are registered as public limited
component of the 'Sagarmala' initiative
companies under the Companies Act,
launched by the Government of India? 2013.
(a) Encouraging coastal economic 2. They accept deposits and lend to un-
served and underserved sections.
development by promoting port based
3. Like commercial banks, these banks
Special Economic Zones have to maintain both Cash Reserve
(b) Efficient evacuation system by Ratio (CRR) and Statutory Liquidity
improving port-hinterland linkages Ratio (SLR).
Which of the statements given above are
(c) Deep Ocean Survey and Exploration correct?
(d) Modernisation of port infrastructure (a) 1 and 2 only
(b) 1 and 3 only
(c) 2 and 3 only
65. Consider the following pairs:
(d) 1, 2 and 3
Country Important reason for
being in the news 68. With reference to the current regulations of
crypto assets in India, consider the following
recently
statements:
1.Turkey : UN changed name of the 1. Consumers availing such assets are
country protected under the Consumer Protection
2.Sudan : Tribal clashes Act, 2019.
2. They are defined as securities under the
3.Zimbabwe : High inflation
Securities Contracts (Regulation) Act,
4.Cambodia : China building foreign 1956.
outpost 3. They are regulated by RBI under the
Foreign Exchange Management Act,
How many pairs given above are correctly
1999.
matched? Which of the statements given above is/are
(a) Only one pair correct?
(a) 3 only
(b) Only two pair
(b) 1 and 3 only
(c) Only three pair (c) 2 only
(d) All four pairs (d) None

13 www.visionias.in ©Vision IAS


69. Which one of the following institutions 72. The Agriculture Ministry recently unveiled
releases the Environmental Performance India’s first Covid-19 vaccine for animals. In
this context, consider the following
Index (EPI)? statements regarding Ancovax:
(a) United Nations Environment Programme 1. It can protect animals against the Delta
and Omicron variants of SARS-CoV-2.
(b) World Economic Forum
2. It is only for the cattle and dogs.
(c) United Nations Development 3. It is an inactivated vaccine developed
Programme using an infectious part of the Delta
variant.
(d) World Wide Fund for Nature
Which of the statements given above is/are
correct?
70. With reference to aggregate monetary (a) 1 only
(b) 1 and 3 only
resources, consider the following statements:
(c) 1 and 2 only
1. M3 is known as narrow money. (d) 2 and 3 only
2. M1 is the least liquid while M4 is the
73. In the context of the Free Trade Agreement
most liquid money measure.
(FTA), the "negative list" includes
Which of the statements given above is/are (a) the list of items which are not permitted
correct? to be exported.
(b) list of products on which duty is to be
(a) 1 only
reduced.
(b) 2 only (c) items of negative externalities like sin
(c) Both 1 and 2 goods.
(d) items on which no concessions would be
(d) Neither 1 nor 2
allowed.

71. In the context of core inflation, consider the 74. Consider the following statements regarding
Agricultural Produce Market Committees
following statements: (APMCs) in India:
1. It takes into account changes in the price 1. These are statutory committees formed
by the state governments.
of food and energy.
2. These committees oversee the trading of
2. When core inflation rises, the Reserve both notified agricultural products as
Bank of India (RBI) increases its key well as livestock products.
3. The committees prohibit the
policy rates to suck excess liquidity from
involvement of any kind of
the market; intermediaries or commissioning agents
Which of the statements given above is/are in the trade between the farmers and
buyers.
correct?
Which of the statements given above is/are
(a) 1 only correct?
(b) 2 only (a) 1 only
(b) 1 and 2 only
(c) Both1 and 2
(c) 1 and 3 only
(d) Neither 1 nor 2 (d) 2 and 3 only

14 www.visionias.in ©Vision IAS


75. Which one of the following statements best 78. With reference to Treasury Bills (T-Bills),
describes Ricardian Equivalence? which one of the following statements is
(a) The market forces change and alter the correct?
exchange rates in order to maintain (a) They are short term capital market
equilibrium. instruments.
(b) Taxation and borrowing are equivalent (b) In India, T- Bills are issued only by the
means of financing expenditure. central government.
(c) Increase in borrowing by the (c) T- Bills are considered as assets of the
government will lead to decreased government.
savings by the consumer. (d) Foreign Portfolio Investment (FPI) is not
(d) Rate of inflation is directly proportional allowed in Treasury Bills.
to the rate of unemployment in the
economy. 79. Consider the following statements regarding
the Stockholm+50 conference:
76. Which of the following can be classified as 1. It is an international meeting convened
short-term external debt of the country? by the United Nations General
1. Foreign Institutional Investors' (FII) Assembly.
investment in corporate securities 2. Stockholm conference was first
2. Investment in Treasury Bills by foreign proposed by Sweden in 1968.
Central Bank 3. Theme of the conference is “a healthy
3. Export Credits planet for prosperity of all - our
4. External commercial borrowings responsibility, our opportunity”.
Select the correct answer using the code Which of the statements given above is/are
given below. correct?
(a) 1, 2, 3 and 4 (a) 1 only
(b) 3 and 4 only (b) 1 and 3 only
(c) 1 and 2 only (c) 2 only
(d) 1 and 4 only (d) 1, 2 and 3

77. Consider the following statements with 80. Consider the following statements regarding
reference to Saint Kabir: the Infrastructure Investment Trust (InvITs):
1. He belonged to the Nirguna tradition of 1. These are mutual fund-like institutions
the Bhakti Movement. that enable investments into the
2. He dismissed the sacred authority of infrastructure sector by pooling small
religious books like Vedas and the sums of money from individual
Quran. investors.
3. He sung compositions known as padas
2. InvITs are regulated by the Securities
and sabdas.
and Exchange Board of India (SEBI).
Which of the statements given above are
Which of the statements given above
correct?
is/are correct?
(a) 1 and 2 only
(a) 1 only
(b) 2 and 3 only
(b) 2 only
(c) 1 and 3 only
(c) Both 1 and 2
(d) 1, 2 and 3
(d) Neither 1 nor 2

15 www.visionias.in ©Vision IAS


81. With reference to depreciation, consider the 84. Which of the following statements regarding
following statements: 'Pradhan Mantri Suraksha Bima Yojana'
1. Depreciation is an annual allowance for is/are correct?
wear and tear of a capital good. 1. It is a life insurance scheme renewable
2. Depreciation does not take into account from year to year offering coverage for
unexpected destruction of capital goods. death due to any reason.
Which of the statements given above is/are 2. Individuals in the age group of 18-70
correct? years having a savings bank or a post

(a) 1 only office account are entitled to enroll

(b) 2 only under the scheme.

(c) Both 1 and 2 3. The scheme is delivered through banks


including regional rural banks as well as
(d) Neither 1 nor 2
cooperative banks.
Select the correct answer using the code
82. Which of the following state-level taxes
given below.
have been subsumed under Goods and
(a) 1 only
Service Tax (GST)?
(b) 1 and 3 only
1. Luxury tax
(c) 2 and 3 only
2. Sales tax
(d) 1, 2 and 3
3. Stamp duty
4. Octroi
85. Which of the following can be patented
Select the correct answer using the code
under the Indian Patent laws?
given below.
(a) An invention relating to atomic energy
(a) 1 and 3 only
(b) Surgical methods
(b) 2, 3 and 4 only
(c) Plants and animals
(c) 1, 2, 3 and 4 (d) Microorganisms
(d) 1, 2 and 4 only

86. Which of the following is/are considered as


83. Consider the following statements: assets of a commercial bank?
1. Saririka stupas are the stupas erected 1. Reserves of commercial banks with the
over the bodily relics of Lord Buddha. Central Bank
2. Lord Buddha's relics from his funeral 2. Deposits of people with the commercial
pyre were collected and given to King bank
Ashoka. 3. Loans given to the public by the
Which of the statements given above is/are commercial bank
correct? Select the correct answer using the code
(a) 1 only given below.
(b) 2 only (a) 1 and 2 only
(c) Both 1 and 2 (b) 2 only
(d) Neither 1 nor 2 (c) 1 and 3 only
(d) 2 and 3 only

16 www.visionias.in ©Vision IAS


87. Consider the following statements with 90. With reference to the Monetary Policy
reference to the BrahMos Missile: Committee (MPC), consider the following
1. It has been developed under the statements:
Integrated Guided Missile Development 1. The committee was set up with the
Programme. responsibility for price stability and
2. It is a two-stage missile with a solid inflation targeting.
propellant booster engine. 2. Its meetings are chaired by the Finance
3. It operates on ‘Fire and Forget Minister.
Principle’, adopting varieties of flights 3. The committee is constituted under the
on its way to the target. Reserve Bank of India Act, 1934.
Which of the statements given above are Which of the statements given above are
correct? correct?
(a) 1 and 2 only
(a) 1 and 2 only
(b) 2 and 3 only
(b) 1 and 3 only
(c) 1 and 3 only
(c) 2 and 3 only
(d) 1, 2 and 3
(d) 1, 2, and 3

88. Consider the following statements with


91. The Ministry of Commerce has identified 12
reference to Fishing Cat:
Champion Service Sectors. Which of the
1. In India, fishing cats are found in the
following are included under the 12 sectors?
mangrove forests of the Sundarbans
1. Tourism and Hospitality
only.
2. Transport and Logistics
2. It is the state animal of West Bengal.
3. Environmental Services
3. It is listed as endangered on the IUCN
4. Biotechnology and pharmaceuticals
Red List of Species.
Select the correct answer using the code
Which of the statements given above are
correct? given below.
(a) 1 and 2 only (a) 1 and 3 only
(b) 2 and 3 only (b) 1, 2, 3 and 4
(c) 1 and 3 only (c) 2 and 4 only
(d) 1, 2 and 3 (d) 1, 2 and 3 only

89. This five-year plan aimed at developing the 92. This biodiverse wetland situated in Iraq,
Indian economy on socialist lines. It also south of the capital Baghdad, has
wanted to put India on a path toward disappeared largely due to human activity
industrialization. The chief architect of this and climate change. It was formed over
plan was P.C. Mahalanobis. limestone rock and studded with gypsum
Which of the following Five Year Plans has formations and has no inlet or outlet. This is
been described in the passage given above? the most likely description of:
(a) First five-year plan (a) Lake Urmia
(b) Second five-year plan (b) Lake Sawa
(c) Fourth five-year plan (c) Lake Tiberias
(d) Sixth five-year plan (d) Sea of Galilee

17 www.visionias.in ©Vision IAS


93. EASE 5.0 ‘Common Reforms Agenda’ 96. Which of the following fall under the capital
sometimes mentioned in the news recently account of the Balance of Payments?
has been developed for 1. Intergovernmental loans
(a) Railways 2. Private remittances
(b) Public Sector Banks 3. Gifts from abroad
(c) Micro, Small and Medium Enterprises 4. Foreign Institutional Investment
(d) Digital payments Select the correct answer using the code
given below.
(a) 1 and 2 only
94. With reference to the different types of
(b) 2 and 3 only
diabetes, consider the following statements:
(c) 3 and 4 only
1. In type 1 diabetes, the body’s insulin
(d) 1 and 4 only
production either goes down or the cells
become resistant to the insulin.
97. Consider the following statements regarding
2. In type 2 diabetes, the pancreas
the Current Account of Balance of
completely stops producing insulin.
Payments:
3. In diabetic ketoacidosis, the human body 1. A country with a surplus on the current
is not able to absorb glucose for energy account necessarily borrows from other
and starts breaking down fats instead. countries.
4. If a person with type 2 diabetes stops 2. A surplus on the current account
taking their insulin, they die within necessarily implies that the country has a
weeks. surplus balance of trade.
Which of the statements given above is/are Which of the statements given above is/are
correct? correct?
(a) 2 and 4 only (a) 1 only
(b) 2, 3 and 4 only (b) 2 only
(c) 3 only (c) Both 1 and 2
(d) 1, 2 and 4 only (d) Neither 1 nor 2

95. In order to calculate Personal Income (PI), 98. With reference to the term ‘spread’ in
which of the following is required to be banking system, which one of the following
statements is correct?
subtracted from the National Income (NI)?
(a) It is the rate charged by the central bank
1. Undistributed Profit
for lending funds to commercial banks.
2. Net interest payments made by
(b) It is the rate at which the central bank of
households
a country lends money to commercial
3. Corporate tax
banks in the event of any shortfall of
4. Transfer payment to the household from
funds.
the government and firms
(c) It is the difference between the interest
Select the correct answer using the code rates of borrowers and depositors in a
given below. bank.
(a) 1 and 4 only (d) It is a specified minimum fraction of the
(b) 1, 2 and 3 only total deposits of customers, which
(c) 2, 3 and 4 only commercial banks have to hold as
(d) 1, 2, 3, and 4 reserves with the central bank.

18 www.visionias.in ©Vision IAS


99. With reference to the factor cost, consider
the following statements:
1. Factor cost includes the payments to the
factors of production only.
2. Factor cost does not include any tax.
Which of the statements given above
is/are not correct?
(a) 1 only
(b) 2 only
(c) Both 1 and 2
(d) Neither 1 nor 2

100. In the context of the 1991 Balance of


Payment crisis, which of the following
was not one of the steps taken under the
New Economic Policy (NEP)?
(a) Lowering of tax rates for better
compliance of the taxpayers
(b) Revaluation (upward adjustment) of
Indian rupee against foreign currencies
(c) Establishment of domestic and foreign
private sector banks in India
(d) Industrial licensing was abolished for
almost all except certain product
categories

19 www.visionias.in ©Vision IAS


VISIONIAS
www.visionias.in
ANSWERS & EXPLANATIONS
GENERAL STUDIES (P) TEST – 3791 (2023)

Q 1.B
• The Market Intervention Scheme (MIS) is a price support mechanism implemented at the request of
state governments for the procurement of perishable and horticultural commodities in the event of a
fall in market prices. Hence statement 1 is not correct.
• Its objective is to protect the growers of these horticultural/agricultural commodities from making distress
sales in the event of a bumper crop. It is implemented when there is at least a 10% increase in
production or a 10% decrease in the ruling rates over the previous normal year.
• Under the MIS, a pre-determined quantity at a fixed Market Intervention Price (MIP) is procured
by NAFED as the central agency. The area of operation is restricted to the concerned state only. Hence
statement 2 is correct.
• Proposal of MIS is approved on the specific request of State/UT Government, if they are ready to bear
50% loss (25% in case of North-Eastern States), if any, incurred on its implementation.
• Under MIS, funds are not allocated to the States. Instead, the central share of losses as per the
guidelines of MIS is released to the State Governments/UTs, for which MIS has been approved based on
specific proposals received from them.

Q 2.D
• Under Article 112 of the Constitution of India, the Annual Financial Statement has to distinguish the
expenditure of the Government on revenue account from other expenditures. The government
Budget, therefore, comprises of Revenue Budget and Capital Budget. The Revenue Budget consists
of the revenue receipts of the Government (tax revenues and other revenues like interest and dividends on
investments made by the Government, fees, and other receipts for services rendered by the Government)
and the expenditure met from these revenues.
• Revenue expenditure does not result in the creation of assets for the Government of India while
Capital Expenditure is related to the creation of Assets, Capital expenditure also includes
investment by the government that yields profits or dividends in the future.
• Revenue Expenditure relates to those expenses incurred for the normal functioning of the government
departments and various services, interest payments on debt incurred by the government, and grants
given to state governments and other parties (even though some of the grants may be meant for the
creation of assets). Pensions and Subsidies, Salaries, and interest payments on market loans form a
part of the revenue expenditure of the Government. Hence options 1, 2 and 4 are correct.
o Effective Revenue Deficit is the difference between revenue deficit and grants for the creation of
capital assets. It signifies the amount of capital receipts that are being used for actual consumption
expenditure of the Government.
• Purchase of new weapons and weapon systems such as missiles, tanks, fighter jets, and submarines
requires extensive capital investment. Nearly a third of the central government’s capital expenditure
goes into the defense sector, mostly for weapon purchases. Hence the purchase of Rafale jets forms a
part of Capital expenditure. Hence option 3 is not correct.

Q 3.D
• Statutory Liquidity Ratio: The statutory liquidity ratio refers to that proportion of total deposits which
the commercial banks are required to keep with themselves in a liquid form. Commercial banks generally
make use of this money to purchase government securities.
1 www.visionias.in ©Vision IAS
• Thus, the statutory liquidity ratio, on the one hand, is used to siphon off the excess liquidity of the banking
system, and on the other, it is used to mobilize revenue for the government. SLR is a tool for controlling
liquidity in the domestic market via manipulating bank credit. Hence statement 2 is correct.
• Marginal Standing Facility: Marginal Standing Facility (MSF) rate refers to the rate at which the
scheduled banks can borrow funds overnight from RBI against government securities. MSF is a very
short-term borrowing scheme for scheduled commercial banks. Banks may borrow funds through MSF
during severe cash shortage or acute shortage of liquidity. Hence statement 1 is correct.
• The MSF is the last resort for banks once they exhaust all borrowing options including the liquidity
adjustment facility by pledging through government securities, which have a lower rate (i.e. repo rate) of
interest in comparison with the MSF.
• The MSF would be a penal rate for banks and the banks can borrow funds by pledging government
securities within the limits of the statutory liquidity ratio. The scheme has been introduced by RBI with
the main aim of reducing volatility in the overnight lending rates in the inter-bank market and to enable
smooth monetary transmission in the financial system.
• MSF, being a penal rate, is always fixed above the repo rate. The MSF would be the last resort for
banks once they exhaust all borrowing options including the liquidity adjustment facility by pledging
government securities, where the rates are lower in comparison with the MSF. Hence statement 3 is
correct.
o MSF represents the upper band of the interest corridor with repo rate at the middle and reverse repo as
the lower band.

Q 4.C
• The Bretton Woods system of monetary management established the rules for commercial and financial
relations among the United States, Canada, Western European countries, Australia, and Japan after the
1944 Bretton Woods Agreement. The Bretton Woods system was the first example of a fully negotiated
monetary order intended to govern monetary relations among independent states
• The Bretton Woods Institutions are the World Bank and the International Monetary Fund
(IMF). They were set up at a meeting of 43 countries in Bretton Woods, New Hampshire, the USA in July
1944. Their aims were to help rebuild the shattered postwar economy and to promote international
economic cooperation.
• The World Bank: The World Bank is an international financial institution that provides loans and grants
to the governments of low- and middle-income countries for the purpose of pursuing capital projects. It is
headquartered in Washington D.C.
• The IMF: The International Monetary Fund (IMF) is an international financial institution, headquartered
in Washington, D.C., consisting of 190 countries. Its stated mission is "working to foster global monetary
cooperation, secure financial stability, facilitate international trade, promote high employment and
sustainable economic growth, and reduce poverty around the world.
• WTO: The World Trade Organization (WTO) is an intergovernmental organization that regulates and
facilitates international trade. Governments use the organization to establish, revise, and enforce the rules
that govern international trade. It is headquartered in Geneva, Switzerland.
• WEF: The World Economic Forum is an international non-governmental and lobbying organization based
in Cologny, canton of Geneva, Switzerland. It was founded on 24 January 1971 by German engineer and
economist Klaus Schwab.
• Hence, option (c) is the correct answer.

Q 5.D
• According to National Commission for Enterprises in the Unorganised Sector, an unorganized
sector refers to production or service-oriented enterprise owned by individuals or self-
employed workers and if workers are employed, then the total number of workers cannot exceed 10.
Central Statistical Organisation uses the term organized enterprise as small units with ten or more
workers with power or 20 or more workers without power for the manufacturing sector. Hence,
statement 1 is not correct.
• The term ‘unorganized worker’ is defined in India under Section 2(m) of the Unorganized Workers
Social Security Act, 2008. An unorganized worker is a home-based worker or a self-employed worker or
a wage worker in the unorganized sector and includes a worker in the organized sector who is not covered
2 www.visionias.in ©Vision IAS
by any of the Acts pertaining to welfare Schemes as mentioned in Schedule II of Unorganized Workers
Social Security Act, 2008. Hence, statement 2 is not correct.
• The unorganized workers are essentially those who do not have the benefit of pension, provident fund,
gratuity, maternity leave etc. and work mostly on daily/hourly wages. They are not represented by active
trade unions.
Q 6.C
• Recent context: Former Twitter CEO Jack Dorsey recently announced his vision for a new
decentralized web platform that is being called Web 5.0 and is being built with an aim to return
“ownership of data and identity to individuals”.
• What do the terms Web 1.0, Web 2.0 and Web 3.0 mean?
o Web 1.0 was the first generation of the global digital communications network. It is often referred to
as the “read-only” Internet made of static web-pages that only allowed for passive engagement.
o Web 2.0 was the “read and write” Internet. Users were now able to communicate with servers and
other users leading to the creation of the social web. This is the World Wide Web that we use today.
o Web 3.0 is an evolving term that is used to refer to the next generation of Internet – a “read-write-
execute” web – with decentralization as its bedrock. It leverages the blockchain technology and will
be driven by Artificial Intelligence and machine learning.
o Web 4.0 services will be autonomous, proactive, content-exploring, self-learning, collaborative, and
content-generating agents based on fully matured semantic and reasoning technologies as well as AI.
They will support adaptive content presentation that will use the Webdatabase via an intelligent
agent. Examples might be services interacting with sensors and implants, natural-language
services, or virtual reality services.
• What is Web 5.0?
o Web 5.0 is aimed at building an extra decentralized web that puts you in control of your data and
identity.
o Simply put, Web 5.0 is Web 2.0 plus Web 3.0 that will allow users to ‘own their identity’ on the
Internet and ‘control their data’. Hence statement 2 is correct.
o Both Web 3.0 and Web 5.0 envision an Internet without threat of censorship – from governments
or big tech, and without fear of significant outages. Hence statement 1 is correct.

Q 7.A
• In WTO terminology, subsidies in general are identified by “boxes” which are given the colours of traffic
lights: green (permitted), amber (slow down — i.e. need to be reduced), red (forbidden). In agriculture,
things are, as usual, more complicated. The Agriculture Agreement has no red box, although domestic
support exceeding the reduction commitment levels in the amber box is prohibited; and there is a blue box
for subsidies that are tied to programmes that limit production.
• Amber Box: Nearly all domestic support measures considered to distort production and trade (with some
exceptions) fall into the amber box. These include measures to support prices, or subsidies directly related
to production quantities.
• Green Box: The green box is defined in Annex 2 of the Agriculture Agreement. In order to qualify, green
box subsidies must not distort trade, or at most cause minimal distortion. They have to be government-
funded (not by charging consumers higher prices) and must not involve price support.
o They tend to be programmes that are not targeted at particular products, and include direct income
supports for farmers that are not related to (are “decoupled” from) current production levels or prices.
They also include environmental protection and regional development programmes.
o Green box” subsidies are therefore allowed without limits, provided they comply with the policy-
specific criteria. Hence, statement 1 is correct.
o India's Public distribution system does not come under the Green Box. Hence, statement 2 is not
correct.
3 www.visionias.in ©Vision IAS
• Blue Box: This is the “amber box with conditions” — conditions designed to reduce distortion. Any
support that would normally be in the amber box, is placed in the blue box if the support also requires
farmers to limit production. At present there are no limits on spending on blue box subsidies.

Q 8.C
• Article 292 of the Indian Constitution states that the Government of India can borrow amounts specified
by the Parliament from time to time. Article 293 of the Indian Constitution mandates that the State
Governments in India can borrow only from internal sources. Thus the Government of India incurs both
external and internal debt, while State Governments incur only internal debt. As per the recommendations
of the 12th Finance Commission, access to external financing by the States for various projects is
facilitated by the Central Government, which provides the sovereign guarantee for these borrowings.
From April 1, 2005, all general category states borrow from multi-lateral and bilateral agencies ( World
Bank, ADB etc.) on a back-to-back basis viz. the interest cost and the risk emanating from currency and
exchange rate fluctuations are passed on to States. In the case of special category states ( North-eastern
states, Himachal, Uttarakhand and J&K), external borrowings of state governments are given by the
Union Government as 90 per cent loan and 10 per cent grant.
• In India, total Central Government Liabilities constitute the following three categories;
[i] Internal Debt.
[ii] External Debt.
[iii] Public Account Liabilities
• Public Debt in India includes only Internal and External Debt incurred by the Central Government.
Internal Debt includes liabilities incurred by resident units in the Indian economy to other resident
units, while External Debt includes liabilities incurred by residents to non-residents.
• The major instruments covered under Internal Debt are as follows:
o Dated Securities: Primarily fixed coupon securities of short, medium and long-term maturity which
have a specified redemption date. These are the single most important component of financing the
fiscal deficit of the Central Government (around 91 % in 2010-11) with an average maturity of around
10 years. Hence option 2 is correct.
o Treasury-Bills: Zero-coupon securities that are issued at a discount and redeemed in face value at
maturity. These are issued to address short term receipt-expenditure mismatches under the auction
program of the Government. These are primarily issued in three tenors, 91,182 and 364 days.
o 14-Day Treasury Bills.
o Securities issued to International Financial Institutions: Securities issued to institutions viz. IMF,
IBRD, IDA, ADB, IFAD etc. for India’s contributions to these institutions etc. Hence option 1 is
correct.
o Securities issued against ‘Small Savings’: All deposits under small savings schemes are credited to
the National Small Savings Fund (NSSF). The balance in the NSSF (net of withdrawals) is invested in
special Government securities.
o Market Stabilization Scheme (MSS) Bonds: Governed by a MoU between the GoI and the RBI,
MSS was created to assist the RBI in managing its sterilization operations. GoI borrows under this
scheme from the RBI, while proceeds from such borrowings are maintained in a separate cash account
with the latter and is used only for redemption of T-bills /dated securities raised under this
scheme. Hence option 3 is correct.
• Gross External Debt, is defined as the outstanding amount of those actual current liabilities, that require
payment(s) of principal and/or interest by the debtor, in the future as per the terms laid out in the contract
between the debtor and the creditor and that are owed to non-residents by the residents of the economy.
• In India, (Gross) External Debt is classified primarily into Long term and Short term:
o Long-Term debt is further classified into (a) Multilateral Debt (b) Bilateral Debt (c) ‘IMF’ signifying
SDR allocations to India by the IMF (c) Export Credit (d) (External) Commercial Borrowings (e) NRI
Deposits and (d) Rupee Debt. Hence option 4 is not correct.

4 www.visionias.in ©Vision IAS


o Short Term Debt is classified into (a) Trade Credits (of up to 6 months and above 6 months and up
to 1 year) (b) Foreign Institutional Investors’ (FII) Investment in Government Treasury-Bills and
Corporate Securities (c) Investment in Treasury-bills by foreign Central Banks and International
Institutions etc. and (iv) External Debt liabilities of the Central Bank and Commercial Banks.

Q 9.B
• Bank rate is the rate at which central bank lends money to the commercial banks by buying their
eligible rated securities - bills of exchange or commercial paper.
• A change in bank rate affects other market rates of interest. An increase in bank rate leads to an increase
in other rates of interest, and conversely, a decrease in bank rate results in a fall in other rates of interest.
Bank rate is also referred to as the discount rate. A deliberate manipulation of the bank rate by the Reserve
Bank to influence the flow of credit created by the commercial banks is known as bank rate policy.
• An increase in bank rate results in an increase in the cost of credit or cost of borrowing. This in
turn leads to a contraction in demand for credit. A contraction in demand for credit restricts the
total availability of money in the economy, and hence loans taken by the commercial banks become
more expensive. Hence statement 1 is not correct.
• The RBI can influence money supply by changing the rate at which it gives loans to the commercial
banks. This rate is called the Bank Rate in India. By increasing the bank rate, loans taken by
commercial banks become more expensive; this reduces the reserves held by the commercial bank and
hence decreases money supply. A fall in the bank rate can increase the money supply. Hence
statement 2 is correct.
o An increase in bank rate leads to decrease in reserves of the banks, which decreases the money supply
in the economy.
o A low (or high) bank rate encourages banks to keep the smaller (or greater) proportion of their
deposits as reserves, since borrowing from RBI is now less (or more) costly than before. As a result,
banks use a greater(or smaller) proportion of their resources for giving out loans to borrowers or
investors, thereby enhancing (or depressing) the multiplier process via assisting(or resisting)
secondary money creation. In short, a low (or high) bank rate reduces(or increases) rdr and hence
increases (or decreases) the value of the money multiplier, which is (1 + cdr)/(cdr + rdr). Thus, for
any given amount of high-powered money, H, total money supply goes up.
• Penal rates are linked with Bank Rates. For instance, if a bank does not maintain the required levels of
CRR and SLR, then RBI can impose a penalty on such banks.
• Nowadays, the bank rate is not used a tool to control money supply, rather LAF (Repo Rate) is used to
control the money supply in the economy.

Q 10.C
• The currency deposit ratio (cdr) is the ratio of money held by the public in currency to that they
hold in bank deposits. cdr = CU/DD. If a person gets Re 1 she will put Rs 1/(1 + cdr) in her bank account
and keep Rs cdr/(1 + cdr) in cash. Hence statement 1 is correct.
• It reflects people’s preference for liquidity. It is a purely behavioral parameter that depends, among
other things, on the seasonal pattern of expenditure. For example, cdr increases during the festive season
as people convert deposits to cash balance for meeting extra expenditures during such periods. Hence
statement 2 is correct.

Q 11.A
• Fiscal Responsibility and Budget Management (FRBM) became an Act in 2003. The objective of the Act
is to ensure inter-generational equity in fiscal management, long-run macroeconomic stability, better
coordination between fiscal and monetary policy, and transparency in the fiscal operation of the
Government.
• FRBM Act provides a legal institutional framework for fiscal consolidation. It is now mandatory for the
Central government to take measures to reduce the fiscal deficit, eliminate revenue deficit and
generate revenue surplus in the subsequent years. The Act binds not only the present government but
also the future Government to adhere to the path of fiscal consolidation. The Government can move away
from the path of fiscal consolidation only in case of natural calamity, national security and other
exceptional grounds that Central Government may specify. Hence statement 2 is not correct.

5 www.visionias.in ©Vision IAS


• Further, the Act prohibits borrowing by the government from the Reserve Bank of India, thereby,
making monetary policy independent of fiscal policy. The Act bans the purchase of primary issues
of the Central Government securities by the RBI after 2006, preventing monetization of government
deficit. The Act also requires the government to lay before the parliament three policy statements in each
financial year namely Medium Term Fiscal Policy Statement; Fiscal Policy Strategy Statement and
Macroeconomic Framework Policy Statement. Hence statement 1 is correct.
• Through Finance Act 2012, amendments were made to the Fiscal Responsibility and Budget Management
Act, 2003 through which it was decided that in addition to the existing three documents, the Central
Government shall lay another document - the Medium Term Expenditure Framework Statement (MTEF) -
before both Houses of Parliament in the Session immediately following the Session of Parliament in
which Medium-Term Fiscal Policy Statement, Fiscal Policy Strategy Statement and Macroeconomic
Framework Statement are laid.
• The concepts of the “Effective Revenue Deficit” and “Medium Term Expenditure Framework”
statement are the two important features of the amendment to the FRBM Act in the direction of
expenditure reforms. Effective Revenue Deficit is the difference between revenue deficit and grants for
the creation of capital assets. This will help in reducing the consumptive component of the revenue deficit
and create space for increased capital spending. Effective revenue deficit has now become a new fiscal
parameter. The “Medium-term Expenditure Framework” statement will set forth a three-year rolling target
for expenditure indicators.

Q 12.D
• Special Drawing Rights (SDR) is an international reserve asset, created by the IMF in 1969 to supplement
its member countries’ official reserves. To date, a total of SDR 660.7 billion (equivalent to about US$943
billion) have been allocated.
• The value of the SDR is based on a basket of five currencies—the U.S. dollar, the euro, the Chinese
renminbi, the Japanese yen, and the British pound sterling.
o Currencies included in the SDR basket have to meet two criteria: the export criterion and the freely
usable criterion.
• The SDR serves as the unit of account of the IMF and other international organizations. The SDR is
neither a currency nor a claim on the IMF. Rather, it is a potential claim on the freely usable currencies
of IMF members. SDRs can be exchanged for these currencies.
• The SDR value in terms of the U.S. dollar is determined daily based on the spot exchange rates
observed at around noon London time. Hence, option (d) is correct.
• The IMF has the authority to prescribe other holders of SDRs, nonmembers, member countries that are
not SDR Department Participants. SDRs cannot be held by private entities or individuals.

Q 13.B
• The National Sample Survey Organization (NSSO) provides three different estimates of employment and
unemployment based on different approaches / reference periods used to classify an individual’s activity
status. These are the
o Usual status approach with a reference period of 365 days preceding the date of survey. Hence
statement 1 is not correct.
o Current weekly status approach with a reference period of seven days preceding the date of survey
o Current daily status approach with each day of the seven days preceding date of survey as the
reference period. Hence statement 2 is correct.
• The Usual Status approach to measuring unemployment uses a reference period of 365 days i.e. one
year preceding the date of the survey of NSSO for measuring unemployment. This approach records only
those persons as unemployed who had no gainful work for a major time during the 365 days preceding the
date of the survey and are seeking or are available for work. Thus, the estimates of unemployment
obtained on the basis of the usual status approach are expected to capture long-term unemployment.
o The usual status approach to measuring unemployment fails to capture the short-term fluctuations in
employment and unemployment caused due to seasonality in labour markets. However, Current

6 www.visionias.in ©Vision IAS


Weekly Status (CWS) measures these short-term fluctuations very well owing to its shorter reference
period of a week. Hence statement 3 is correct.
• The Current Weekly Status (CWS) approach to measuring unemployment uses seven days preceding
the date of the survey as the reference period.
o A person is considered to be employed if he or she pursues any one or more of the gainful activities
for at least one-hour on any day of the reference week. On the other hand, if a person does not pursue
any gainful activity, but has been seeking or available for work, the person is considered as
unemployed.
• The current daily status approach to measuring unemployment seeks to ascertain the activity status of an
individual for each day of the reference week. It reports time disposition of an individual on each day of
the reference week. This means that in addition to recording the activity being pursued, time-intensity is
also recorded in quantitative terms for each day of the reference week.

Q 14.D
• Recent Context: While setting aside National Green Tribunal (NGT) ban on construction at
Rushikonda Hills in Visakhapatnam, Supreme Court said that NGT is subordinate to the High
Court in so far as territorial jurisdiction is concerned.
• While High Courts are established by the Constitution, Tribunals are established under the Constitution,
Acts of Parliament or State Legislatures. Hence, statement 1 is not correct.
• While High Courts consist of judicial members only, Tribunals consist of a mix of judicial members and
experts with special knowledge. Hence, statement 2 is not correct.
• Tribunals are limited to the laws under which they are set while High Courts have the power to use all
the enacted laws before making a decision. Hence, statement 3 is not correct.

Q 15.C
• The Local Area Banks (LABs) are small private banks, conceived as low cost structures which would
provide efficient and competitive financial intermediation services in a limited area of operation, i.e.,
primarily in rural and semi-urban areas, comprising three contiguous districts.
• LABs were set up to enable the mobilization of rural savings by local institutions and, at the same
time, to make them available for investments in the local areas. Hence statement 1 is correct.
• Since LABs are being set up in district towns, their activities are focused on the local customers with
lending primarily to agriculture and allied activities, small scale industries, agro-industrial activities,
trading activities and the non-farm sector.
• LABs are also required to observe the priority sector lending targets at 40% of net bank credit (NBC) as
applicable to other domestic banks such as scheduled commercial banks. Hence statement 2 is correct.
o Within the above target, these banks will adhere to the requirement of lending at least 25% of their
priority sector deployments (10% of NBC) to the weaker sections.
• In 2014, RBI has permitted LABs to be converted into small finance banks subject to them meeting the
prescribed eligibility criteria.

Q 16.C
• Recently, a delegation of MLAs from Gujarat visited the Uttar Pradesh Legislative Assembly to
learn about the novel e-Vidhan system for paperless proceedings that has been recently adopted by
the UP state assembly.
• e-Vidhan is a Mission Mode Project to digitize and make the functioning of State Legislatures
paperless. This is part of Digital India programme and the Ministry of Parliamentary Affairs, being the
Nodal Ministry for this project, desires to roll out e-Vidhan as NeVA covering all 40 Houses including
two Houses of Parliament and thereby putting all them on a single platform and proving the theory of
‘One Nation One Application’. Hence, statement 2 is correct.
• NeVA is a work-flow system deployed on NIC Cloud, MeghRaj which helps the Chair of the House to
conduct the proceedings of the House smoothly, Members to carry out their duties in the House efficiently
and to conduct Legislative Business of the House in a Paperless manner. Hence, statement 1 is correct.

7 www.visionias.in ©Vision IAS


• NeVA is a device neutral and member centric application created to equip them to handle diverse House
Business smartly by putting entire information regarding member contact details, rules of procedure, list
of business, notices, bulletins, bills, starred/unstarred questions and answers, papers laid, committee
reports etc. in their hand held devices/ tablets and equip all Legislatures/ Departments to handle it
efficiently. NeVA will completely eliminate the process of sending out a notice/request for the
collection of data.

Q 17.D
• Recent Context: Organization of Islamic Cooperation (OIC) and the six-member Gulf Cooperation
Council (GCC) strongly opposed the remarks defaming Islam and the Prophet Muhammad by the
now-suspended and expelled BJP leaders Nupur Sharma and Naveen Kumar Jindal.
• Over the five years from 2017 through 2021, Iran and the GCC member states comprising the UAE, Saudi
Arabia, Bahrain, Oman, Kuwait and Qatar accounted for a 15.3% share of India's cumulative two-way
merchandise trade of $3.98 trillion in that period, according to statistics available on the UN Comrade
database.
o Of the $609 billion of exports and imports that the seven countries accounted for in this period,
the UAE contributed the lion's share of $277.4 billion, or almost 7%, making it one of India's
largest trading partners. Saudi Arabia was next with $153 billion. Hence statement 2 is not
correct.
• India’s dependence on Gulf countries for its energy needs:
o Observer Research Foundation (ORF) analysed that the share of Gulf countries of India’s total crude
oil imports has remained fairly stable over the past 15 years at around 60%. Hence statement 1
is not correct.
o The ORF study found that in 2020-21, India’s top oil exporter was Iraq with a share of over 22%,
followed by Saudi Arabia – at around 18%.
o UAE, Kuwait and Oman were other Gulf countries among the top-10 suppliers of crude oil to India in
2020-21.

Q 18.C
• According to Article 112 of the Indian Constitution, the Union Budget of a year is referred to as the
Annual Financial Statement. It is a statement of the estimated receipts and expenditure of the
Government in a financial year (which begins on 01 April of the current year and ends on 31 March of the
following year). In addition to it, the Budget contains:
o Estimates of revenue and capital receipts,
o Ways and means to raise the revenue,
o Estimates of expenditure,
o Details of the actual receipts and expenditure of the closing financial year and the reasons for any
deficit or surplus in that year, and
o The economic and financial policy of the coming year, i.e., taxation proposals, prospects of revenue,
spending program, and introduction of new schemes/projects.
• Fiscal Deficit: It is the gap between the government’s expenditure requirements and its receipts. This
equals the money the government needs to borrow during the year. A surplus arises if receipts are more
than expenditures.
Fiscal Deficit = Total expenditure – (Revenue receipts + Non-debt creating capital receipts)
• From the financing side: Gross fiscal deficit = Net borrowing at home + Borrowing from RBI +
Borrowing from abroad. The gross fiscal deficit is a key variable in judging the financial health of the
public sector and the stability of the economy.
• Gross Primary Deficit is Gross Fiscal Deficit minus Net interest liabilities. Net Primary Deficit is Net
Fiscal Deficit minus net interest payments. The net interest payment is interest paid minus interest
receipt. A shrinking primary deficit indicates progress towards fiscal health. Hence statement 2 is
correct.
• Effective Revenue deficit is a term introduced in the Union Budget 2011-12. While revenue deficit is
the difference between revenue receipts and revenue expenditure, the present accounting system includes
8 www.visionias.in ©Vision IAS
all grants from the Union Government to the state governments/Union territories/other bodies as revenue
expenditure, even if they are used to create assets. Such assets created by the sub-national
governments/bodies are owned by them and not by the Union Government. Nevertheless, they do result in
the creation of durable assets; an effective revenue deficit excludes those revenue expenditures (or
transfers) in the form of grants for the creation of capital assets.
• In short, Effective Revenue Deficit is the difference between revenue deficit and grants for the
creation of capital assets. Effective Revenue Deficit signifies that amount of capital receipts that are
being used for actual consumption expenditure of the Government. Hence statement 1 is correct.

Q 19.C
• Prime Minister Narendra Modi is going to inaugurate the Sant Tukaram Shila Mandir in the
temple town of Dehu in Pune district.
• Sant Tukaram Maharaj was a 17th-century Marathi poet and Hindu Saint.
• He worshipped the famous deity of Vishnu known as Vitthala (or Vitobha) in
Pandhapur, Maharastra. He was part of an important Vaishnava tradition known as Dasa Kuta and which
is still influential today. Other poet saints such as Namdev (1270–1350) also came in this line.
• The Bhakti saint Sant Tukaram had sat on this piece of rock for 13 continuous days when challenged
about the authenticity of the Abhangas he had written.
• Sant Tukaram and his work are central to the Warkari sect spread across Maharashtra. His
message about a casteless society and his denial of rituals had led to a social movement. Sant
Tukaram is credited with starting the Wari pilgrimage.
• The Wari sees lakhs of devotees congregating in the temple towns of Dehu and Alandi to accompany the
padukas of Sant Tukaram and Sant Dyaneshwar respectively as they start for Pandharpur. Participants
finish their sowing before they set off. The pilgrims reach Pandharpur on the day of Ekadashi with the
Chief Minister performing the mahapuja.
• Hence, option (c) is the correct answer.

Q 20.A
• Recent Context: India looks forward to joining the NSG, says External Affairs Minister Jaishankar.
• Multilateral Export Control Regimes (MECR) are voluntary and non-binding agreements created by the
major supplier countries that have agreed to co-operate in their effort to prevent and regulate the transfer
of certain military and dual use technology. It aims at preventing the proliferation of Weapons of Mass
Destruction.
o They are independent of the United Nations.
o Their regulations apply only to members and it is not obligatory for a country to join.
o India is now a member of three of the four MECRs, except the Nuclear supplier Group. Hence
option (a) is the correct answer.
• Wassenaar Arrangement:
o The Wassenaar Arrangement is a voluntary export control regime. The Arrangement, formally
established in July 1996, has 42 members who exchange information on transfers of conventional
weapons and dual-use goods and technologies.
▪ Dual-use refers to the ability of a good or technology to be used for multiple purposes - usually
peaceful and military.
o Wassenaar Arrangement’s Secretariat is in Vienna, Austria.
o India was inducted to the Wassenaar Arrangement on 7 December, 2017 as the 42nd member.
• Nuclear Suppliers Group:
o The Nuclear Suppliers Group (NSG) is a group of nuclear supplier countries that seeks to contribute
to the non-proliferation of nuclear weapons through the implementation of two sets of Guidelines for
nuclear exports and nuclear-related exports.
o The NSG came into being as a response to the 1974 nuclear tests by India.
o It has 48 participating governments.
o India is not a member of the NSG because all its efforts were consistently blocked by China and some
other members.
9 www.visionias.in ©Vision IAS
o India’s bid for membership being blocked on the ground of India being a non-signatory to the nuclear
non-proliferation treaty.
• Australia Group:
o It is an informal association of member states that aims to minimize risk of proliferation of chemical
and biological weapons.
o The formation of the Australia Group (AG) in 1985 was prompted by Iraq’s use of chemical weapons
during the Iran-Iraq War (1980-1988).
o India joined the Australia Group (AG) on 19 January 2018.
• Missile Technology Control Regime (MTCR):
o It is an informal political understanding among states that seek to limit proliferation of missiles and
missile technology.
o Formed in 1987 by G-7 countries.
o It is not legally binding.
o India was inducted into the Missile Technology Control Regime in 2016 as the 35th member.

Q 21.C
• The expenditure method is a system for calculating gross domestic product (GDP) that combines
consumption, investment, government spending, and net exports. It is the most common way to estimate
GDP.
• In an economy, there are three main agencies, which buy goods and services. These are: Households,
Firms, and the Government.
o It says everything that the private sector, including consumers and private firms, and government
spend within the borders of a particular country, must add up to the total value of all finished goods
and services produced over a certain period of time.
• This final expenditure is made up of the sum of 4 expenditure items, i.e GDP= C+I+G+X-M namely:
o Consumption (C): Personal Consumption made by households, the payment of which is paid by
households directly to the firms which produced the goods and services desired by the
households. Hence option 1 is correct.
o Investment Expenditure (I): Investment is an addition to the capital stock of an economy in a given
time period. It includes capital expenditures by firms on assets, such as equipment, production
facilities, and plants.
▪ It is to be noted that final investment includes investment on capital goods and not on
intermediate goods. Hence option 2 is not correct and option 3 is correct.
o Government Expenditure (G): It represents expenditures by the government on defense and non-
defense goods and services, such as weaponry, health care, and education.
▪ Government expenditure on pension schemes, scholarships, unemployment allowances etc.
are not included in this as all of them come under transfer payments. Hence option 4 is not
correct.
o Net Exports (X-IM): Expenditure on foreign-made products (Imports) are expenditure that escapes
the system, and must be subtracted from total expenditures. In turn, goods produced by domestic
firms which are demanded by foreign economies involve expenditure by other economies on our
production (Exports), and are included in total expenditure. The combination of the two gives us Net
Exports.

Q 22.D
• The World Bank Group is one of the world’s largest sources of funding and knowledge for developing
countries. Its five institutions share a commitment to reducing poverty, increasing shared prosperity, and
promoting sustainable development. The Five institutions include
o International Bank for Reconstruction and Development (IBRD): The world’s largest
development bank, IBRD provides financial products and policy advice to help countries reduce
poverty and extend the benefits of sustainable growth to all of their people.
o International Development Association (IDA): It is the part of the World Bank that helps the
world’s poorest countries. Established in 1960, IDA aims to reduce poverty by providing zero to
low-interest loans (called “credits”) and grants for programs that boost economic growth,

10 www.visionias.in ©Vision IAS


reduce inequalities, and improve people’s living conditions. Hence, pair 2 is not correctly
matched.
o International Finance Corporation (IFC): IFC, a member of the World Bank Group, advances
economic development and improves the lives of people by encouraging the growth of the private
sector in developing countries. Hence, pair 1 is not correctly matched.
o Multilateral Investment Guarantee Agency (MIGA): MIGA's goal is to promote foreign direct
investment into developing countries to support growth. Hence, pair 3 is correctly matched.
o International Centre for Settlement of Investment Disputes (ICSID): ICSID is the world’s leading
institution devoted to international investment dispute settlement. It has extensive experience in this
field, having administered the majority of all international investment cases.

Q 23.B
• The GDP measures the aggregate production of final goods and services taking place within the
domestic economy. It refers to total market value of all the final goods and services produced in an
economy in a given period of time. For India, this time period is from 1st April to 31st March.
• This means it measures the value of final goods and services produced within a geographic boundary
regardless of the nationality of the individual or firm. Therefore, goods and services produced by a
foreign national within the territory of India, is also calculated in GDP. Hence statement (b) is the
correct answer.
• In GDP, only the final output of such goods and services are included.
• The rule that only finished or final goods must be counted is necessary to avoid double or triple counting
of raw materials, intermediate products, and final products. For example, the value of automobiles already
includes the value of the steel, glass, rubber, and other components that have been used to make them.
• As the GDP takes into account only the goods and services produced within the domestic territory, the
goods and services produced by Indian citizens in foreign territory is not included.
• GDP does not take into account externalities such as pollution from refinery of crude oil.
Externalities refer to the benefits (or harms) a firm or individual causes to another for which they are not
paid (or penalized).
o Externalities do not have any market in which they can be bought and sold. For example, let us
suppose there is an oil refinery that refines crude petroleum and sells it in the market. The output of
the refinery is the amount of oil it refines. We can estimate the value-added of the refinery by
deducting the value of intermediate goods used by the refinery (crude oil in this case) from the value
of its output. The value-added of the refinery will be counted as part of the GDP of the economy. But
in carrying out the production the refinery may also be polluting the nearby river. This may cause
harm to the people who use the water of the river.
o Hence their utility will fall. Pollution may also kill fish or other organisms of the river on which fish
survive. As a result, the fishermen of the river may be losing their income and utility. Such harmful
effects that the refinery is inflicting on others, for which it does not have to bear any cost, are called
externalities.
o In this case, the GDP is not taking into account such negative externalities. Therefore, if we take GDP
as a measure of the welfare of the economy we shall be overestimating the actual welfare. This was an
example of a negative externality. There can be cases of positive externalities as well. In such cases,
GDP will underestimate the actual welfare of the economy.

Q 24.A
• The Green Revolution refers to the large increase in the production of food grains resulting from
the use of high-yielding variety (HYV) seeds, especially for wheat and rice. In India, the Green
Revolution was mainly led by M.S. Swaminathan. Hence statement 2 is correct.
• The use of these HYV seeds required the use of fertilizer and pesticide in the correct quantities as
well as a regular supply of water (reliable irrigation facilities); the application of these inputs in
correct proportions is vital. The farmers who could benefit from HYV seeds required reliable irrigation
facilities as well as the financial resources to purchase fertilizer and pesticides. As a result, in the first

11 www.visionias.in ©Vision IAS


phase of the green revolution (the approximately mid-1960s up to the mid1970s), the use of HYV seeds
was restricted to the more affluent states such as Punjab, Andhra Pradesh, and Tamil Nadu.
• As per famous economist C.H. Hanumantha Rao, a good proportion of the rice and wheat produced
during the green revolution period (available as marketed surplus) was sold by the farmers in the market.
As a result, the price of food grains declined relative to other items of consumption. Hence statements
1 and 3 are not correct.
• The government provided loans at a low-interest rate to small farmers and subsidized fertilizers so
that small farmers could also have access to the needed inputs. Since the small farmers could obtain the
required inputs, the output on small farms equaled the output on large farms in the course of time. As
a result, the green revolution benefited small as well as rich farmers.

Q 25.C
• The Gross Domestic Product (GDP) deflator is a measure of general price inflation. It is calculated by
dividing nominal GDP by real GDP and then multiplying by 100. Therefore, it is a ratio of nominal
GDP to the real GDP. Hence statement 1 is correct.
o GDP Deflator = Nominal GDP/Real GDP x 100
• Nominal GDP is the market value of goods and services produced in an economy, unadjusted for inflation
(It is the GDP measured at current prices). Real GDP is nominal GDP, adjusted for inflation to reflect
changes in real output (It is the GDP measured at constant prices). Therefore, it indicates the change in
prices of commodities from the base year to the current year.
o For example, suppose a country only produces bread. In the year 2000, it had produced 100 units of
bread, the price was Rs 10 per bread.GDP at the current price was Rs 1,000. In 2001 the same country
produced110 units of bread at a price of Rs 15 per bread. Therefore nominal GDP in 2001 was Rs
1,650 (=110 × Rs 15). Real GDP in 2001 calculated at the price of the year 2000 (2000 will be called
the base year) will be 110 × Rs 10 = Rs 1,100.
o The GDP deflator is1,650/1,100 = 1.50 (in percentage terms this is 150 per cent). This implies that the
price of bread produced in 2001 was 1.5 times the price in 2000. Which is true because the price of
bread has indeed gone up from Rs 10 to Rs 15.
• GDP deflator reflects the prices of all domestically produced goods and services in the economy whereas,
other measures like CPI and WPI are based on a limited basket of goods and services, thereby not
representing the entire economy (the basket of goods is changed to accommodate changes in consumption
patterns, but after a considerable period of time).
• CPI and WPI) may differ from GDP deflators because
o The goods purchased by consumers do not represent all the goods that are produced in a country.
GDP deflator takes into account all such goods and services.
o CPI includes prices of goods consumed by the representative consumer, hence it includes prices of
imported goods. GDP deflator does not include prices of imported goods.
o The weights are constant in CPI – but they differ according to the production level of each good in the
GDP deflator. Hence statement 2 is correct.
o Changes in consumption patterns or the introduction of new goods and services or structural
transformation are automatically reflected in the deflator which is not the case with other inflation
measures.
• However, WPI and CPI are available on a monthly basis whereas GDP deflator comes with a lag (yearly
or quarterly, after quarterly GDP data is released). Hence, the monthly change in inflation cannot be
tracked using a GDP deflator, limiting its usefulness.
Q 26.C
• Labor force (also called work force) is the total number of people employed or seeking employment in a
country or region. One is classified as ‘not in labor force’, if he or she was engaged in a relatively longer
period in any one of the non-gainful activities.
• The unemployment rate is defined as the percentage of persons unemployed among the persons in
the labor force. Hence statement 1 is correct.
• It is calculated as follows:
o Unemployment rate = (Unemployed Workers / Total labor force) X 100
• The Worker participation rate is defined as the percentage of total workers (main and marginal) to
the total population. Hence statement 2 is correct.
• It is calculated as follows:
o Work participation rate = (Total Workers / Total Population) X 100
12 www.visionias.in ©Vision IAS
.
Q 27.A
• Recent context: The Indian Farmers Fertiliser Cooperative (IFFCO) has started commercial production of
‘nano urea liquid’, a first-of-its-kind product.
• Liquid nano Urea is a patented chemical nitrogen fertiliser developed by IFFCO’s Nano
Biotechnology Research Centre at Kalol with nano nitrogen particles. Hence statement 3 is correct.
o Nano urea is bio available to plants because of its desirable particle size about 20-50 nm and more
surface area (10,000 times over 1 mm urea prill) and number of particles (55,000 nitrogen particles
over 1 mm urea prill). Hence, Nano Urea increases its availability to crop by more than 80% resulting
in higher Nutrient Use efficiency. Hence statement 2 is correct.
o One nanometre is equal to a billionth of a metre.
o It is sprayed directly on the leaves of plants and gets absorbed by the stomata-pores found on the
epidermis of leaves.
• Benefits of Nano urea:
o Improved agricultural produce with reduced urea consumption as the efficiency of liquid nano urea is
as high as 85-90% in comparison to 25% of conventional urea.
o Significant reduction in urea imports, government subsidies and logistics cost.
o Reduced soil, water and air pollution from urea, improving quality of underground water and
reduction in global warming. Hence statement 1 is correct.
o Higher shelf life as no issue of caking on coming into contact with moisture.
• Other Potential Nanotechnology Opportunities in Agriculture
o Use of Nanoformulations of herbicides, pesticides and other fertilizers.
o Use of Nanosensors to identify diseases and residues of agrochemicals.
o For genetic improvement of plants to increase productivity, nutritional value or shelf life.
Q 28.A
• Fiscal policy is the use of government revenue collection (mainly taxes but also non-tax revenues such as
divestment, loans) and expenditure (spending) to influence the economy. Through fiscal policy, the
government of a country controls the flow of tax revenues and public expenditure to navigate the
economy. If the government receives more revenue than it spends, it runs a surplus, while if it spends
more than the tax and non-tax receipts, it runs a deficit.
• Contractionary fiscal policy is when the government either cuts spending or raises taxes. It gets its
name from the way it contracts the economy. It reduces the amount of money available for businesses and
consumers to spend.
• This policy involves raising taxes or cutting government spending, so that (Government spending <
Tax revenue) it cuts upon the aggregate demand (thus, economic growth) and reduces the
inflationary pressures in the economy.
• The following can be potential traits of Contractionary Fiscal Policy as they aid in increasing the revenue
of the government:
o Decreased expenditure in infrastructure development such as the building of schools, hospitals
o Decrease in Subsidies and pensions
o Increased Tax rates (specially on sin goods, so as to increase the revenue available)
o Cutting incentives of employees to work, stagnation of wages
• All these traits and methods, involve shrinking or contracting the economy, the money left in the hands of
the consumer decreases, leading to decrease in the Purchasing power, which theoretically leads to reduced
inflation.
• Hence option (a) is the correct answer

Q 29.B
• The Reserve Bank of India or RBI mandates that banks store a proportion of their deposits in the form of
cash so that the same can be given to the bank’s customers if the need arises.
• The percentage of cash required to be kept in reserves, vis-a-vis a bank’s total deposits, is called the
Cash Reserve Ratio. The cash reserve is kept with the RBI. Hence statement 1 is not correct.
• Banks do not get any interest on the money that is with the RBI under the CRR requirements. The
percentage of the CRR is decided by the RBI.
13 www.visionias.in ©Vision IAS
.
o Unlike Statutory Liquidity Ratio or SLR, which can be maintained in either gold or cash, CRR needs
to be maintained only in cash.
o CRR helps in keeping inflation under control. At the time of high inflation in the economy, RBI
increases the CRR, so that banks need to keep more money in reserves so that they have less money to
lend further.
• As per the RBI Act 1934, all Scheduled Commercial Banks (that includes public and private sector banks,
foreign banks, regional rural banks and co-operative banks) are required to maintain a cash balance on
average with the RBI on a fortnightly basis to cater to the CRR requirement.
o Non Bank Financial Corporations (NBFCs) are outside the purview of this reserve
requirement. Hence statement 2 is correct.

Q 30.B
• Foreign Exchange Rate (also called Forex Rate) is the price of one currency in terms of another. It links
the currencies of different countries and enables the comparison of international costs and prices. For
example, if we have to pay Rs 50 for $1 then the exchange rate is Rs 50 per dollar.
• People demand foreign exchange because they want to purchase goods and services from other countries;
they want to send gifts abroad; and, they want to purchase financial assets of a certain country.
• A rise in the price of foreign exchange will increase the cost (in terms of rupees) of purchasing a
foreign good. This reduces demand for imports and hence the demand for foreign exchange also
decreases, other things remaining constant.
• A rise in the price of foreign exchange will reduce the foreigner’s cost (in terms of USD) while
purchasing products from India, other things remaining constant. This increases India’s
exports and hence the supply of foreign exchange may increase.
• Hence, option (b) is the correct answer.

Q 31.B
• Consumer Price Index is a measure of change in retail prices of goods and services consumed by
defined population group in a given area with reference to a base year. Hence statement 1 is not
correct.
• This basket of goods and services represents the level of living or the utility derived by the consumers at
given levels of their income, prices and tastes.
• The CPI includes the prices of goods consumed by representative customers, hence it includes
prices of imported goods. Hence statement 2 is correct.
• The consumer price index measures change only in one of the factors i.e. prices. This index is an
important economic indicator and is widely considered as a barometer of inflation, a tool for monitoring
price stability and as a deflator in national accounts.
• Consumer price index is used as a measure of inflation in around 157 countries. The dearness allowance
of Government employees and wage contracts between labour and employer is based on this index.
• In 2011 the CSO brought out a revised CPI, which was CPI (Urban), CPI (Rural) and CPI (Urban +Rural)
with 2010 as the base price. CSO revised the base year of this newly set up index to 2012 in January 2015.

Q 32.C
• The Union Budget, which is presented by the Finance Minister of India, comprises Capital Budget and
Revenue Budget. The Capital Budget is further categorized into capital receipts and capital expenditure.
• Capital receipts are receipts that create liabilities or reduce financial assets. They also refer to
incoming cash flows. Capital receipts can be both non-debt and debt receipts.
• All capital receipts are tax-free unless there is a proviso to tax them. Capital receipts can be both non-debt
and debt receipts.
o Non-debt capital receipts are those which do not incur any future repayment burden for the
government. Examples of non-debt capital receipts: Recovery of loans and advances, disinvestment,
issue of bonus shares, etc.
o Debt Receipts have to be repaid by the government. Around 25 percent of government expenditure is
financed through borrowing. A reduction in debt receipt (or borrowing) can be a big leap for the
14 www.visionias.in ©Vision IAS
economy's financial health. Most of the capital receipts of the government are debt receipts. Examples
of debt capital receipts: Market loans, issuance of special securities to public-sector banks, etc.
• Capital Receipts include:
o loans raised by Government from the public called market loans,
o borrowings by the Government from Reserve Bank and other parties through the sale of
Treasury Bills,
o loans received from foreign Governments and bodies,
o disinvestment receipts
o recoveries of loans from State and Union Territory Governments and other parties.
• Collection and Appropriation of Direct Taxes is a part of Revenue Receipts.
• Hence option (c) is the correct answer.

Q 33.B
• Money multiplier is the ratio of the total money supply to the stock of high-powered money in an
economy. Hence option (b) is the correct answer.
o By definition, money supply is equal to currency plus deposits
▪ M = CU + DD = (1 + cdr )DD
▪ where, cdr = CU/DD
o High-powered money then consists of currency held by the public and reserves of the commercial
banks, which include vault cash and banks’ deposits with RBI.
▪ H = CU + R = cdr.DD + rdr.DD = (cdr + rdr)DD
o Thus the ratio of money supply to high powered money
▪ M/H = 1+cdr/cdr+rdr > 1, as rdr > 1
o This is precisely the measure of the money multiplier.
• The factors affecting the Money multiplier are the: Reserve ratio (SLR, CRR) and banking habits
of the population, etc.
o For instance, assume that X bank has received a deposit of Rs 1000 and both SLR and CRR are
maintained at 10 percent respectively i.e 20 percent in total. Now, the bank will keep Rs 200 as
reserves (SLR and CRR) and the rest of the amount will be made available to the public in the form of
loans. Now, a borrower takes a loan of Rs 800 from the bank either for consumption or for investment
purposes.
o Suppose, the borrower has spent the loan taken for the purchase of an article. The seller of the article
will receive the money and simultaneously deposit Rs 800 again with the bank. This happens because
we have assumed that there is only a single bank in the economy. After receiving Rs 800 from the
seller, the bank will again keep aside 20% of the amount i.e Rs 160 as reserves, and provide a loan to
the public with the remaining amount. This process continues till the initial deposit of Rs 1000
becomes Rs 5000 i.e. 5 times the initial deposit.
• Reserve deposit ratio is the fraction of their total deposits which commercial banks keep as reserves.
• Speculative demand is the demand for money as a store of wealth.
• Transaction demand is the demand for money for carrying out transactions.

Q 34.B
• Recently, the navies of India and Indonesia began a coordinated patrol in the Andaman Sea and
Malacca Strait. The 38th edition of the exercise IND-INDO CORPAT, was conducted for the first
time after the COVID-19 pandemic.
• The Strait of Malacca is a narrow stretch of water between the Malay Peninsula and the Indonesian island
of Sumatra. As the main shipping channel between the Indian and Pacific oceans, it is one of the most
important shipping lanes in the world.
• It connects the Andaman Sea (Indian Ocean) with the South China Sea (Pacific Ocean).
• It is crucial to the flow of global trade as it provides the shortest sea route between Indian Ocean and
Pacific Ocean. Roughly a quarter of all oil transported by sea passes through the straits.
• Hence, option (b) is the correct answer.

15 www.visionias.in ©Vision IAS


Q 35.C
• In India, the minerals are classified as minor minerals and major minerals.
• According to section 3(e) of the Mines and Minerals (Development and Regulation) Act, 1957
“Minor Minerals” means building stones, gravel, ordinary clay, ordinary sand other than sand used
for prescribed purposes, and any other mineral which the Central Government may, by notification in
the Official Gazette, declare to be a minor mineral. (For the purposes of this Act, the word "minerals”
includes all minerals except mineral oils- natural gas and petroleum).
• Major minerals are those specified in the appendices to the Mines and Minerals (Development and
Regulation) Act, 1957 (MMDR Act 1957) and the common major minerals are Major Mineral are
minerals like Agate, Asbestos, Barytes, Bauxite, Cadmium, Coal. Copper, Gold, Iron ore, Lead,
Lignite, Manganese, Nickel, Rock Phosphate, Tungsten, Uranium, Wollastonite, Zinc, etc. It may be
noted that there is no official definition for “major minerals” in the MMDR Act. Hence, whatever is
not declared as a “minor mineral” may be treated as a major mineral.
• The major-minor classification has nothing to do with the availability (abundance or scarcity) of
these minerals, though it is correlated with the relative value of these minerals. Further, this
classification is based more on their end-use, rather than the level of production, level of mechanization,
export, and import, etc. (eg. Sand can be a major mineral or a minor mineral depending on where it is
used; the same is the case for limestone).
• Hence option (c) is the correct answer.

Q 36.A
• The passage given above depicts a scenario of structural unemployment. Structural Unemployment is
a type of unemployment that can last for many years and can be caused by changes in technology or
shifting demographics. Structural unemployment represents a long-term shift in how an economy
functions, leading workers to become marginalized.
• The causes of structural unemployment can include shifts in the economy, improvements in
technology, and workers lacking the job skills that are required for them to find employment.
• Hence option (a) is the correct answer.

Q 37.D
• The process of moving from self-employment and regular salaried employment to casual wage work
is called casualization of the workforce. Casual workers are defined as those who work for others in
farm or non-farm enterprises and are paid wages that are daily or periodic in nature. All daily wage-
earning employees and some categories of contract employees are casual laborers. Hence, option (d) is
the correct answer.
• The wage-paid labor is largely non-unionized due to the casual and seasonal nature of employment and
the scattered location of enterprises. This sector is marked by low incomes, unstable and irregular
employment, and a lack of protection either from legislation or trade unions.
• Formalization of the workforce refers to the situation wherein there is a continuous increase in the
percentage of the workforce in the formal sector and a simultaneous decline in the percentage of the
workforce in the informal sector. Around 90 percent workforce in India is in the ―unregulated informal
sector.

Q 38.C
• MSME stands for Micro, Small, and Medium Enterprises. It was introduced by the Government of India
in agreement with the MSMED (Micro, Small, and Medium Enterprises Development) Act of
2006. As per this act, MSMEs are the enterprises involved in the processing, production, and preservation
of goods and commodities.
• On 13th May 2020, Finance Minister Nirmala Sitharaman added the additional principle of turnover
along with the Investment in Plant & Machinery/equipment to be used as composite criteria for
classification of MSMEs in India.
• Manufacturing Enterprises and Enterprises rendering Services are classified as:

16 www.visionias.in ©Vision IAS


o Micro: Investment in Plant and Machinery or Equipment - Not more than Rs.1 crore and Annual
Turnover ; not more than Rs. 5 crore.
o Small: Investment in Plant and Machinery or Equipment - Not more than Rs.10 crore and Annual
Turnover ; not more than Rs. 50 crore.
o Medium: Investment in Plant and Machinery or Equipment - Not more than Rs.50 crore and Annual
Turnover ; not more than Rs. 250 crore.
• Hence, option (c) is the correct answer.

Q 39.C
• Recent context: 118th meeting of the India-Pakistan Permanent Indus Commission (PIC) was
conducted.
• PIC is a communication channel created under Indus Water Treaty (IWT) for the governments to gather
and resolve questions about the treaty's implementation.
o It is required to meet at least once in a year.
o IWT was signed in 1960 by India and Pakistan with the World Bank as a third-party guarantor.
Hence option (c) is the correct answer.
o It is a water-distribution treaty and delimits the rights and obligations of both countries concerning the
use of waters on the Indus River System.
o It consists of six main rivers with numerous tributaries.
o Under this, waters of three eastern rivers namely Ravi, Beas and Sutlej, have been allocated
exclusively to India.
o Western rivers, namely Indus, Chenab and Jhelum, are for Pakistan with India given some rights
like agriculture, navigation, domestic use and also right to generate hydroelectricity through run of the
river projects.
o IWT does not have a unilateral exit provision, and is supposed to remain in force unless both the
countries ratify another mutually agreed pact.

Q 40.A
• Foreign Portfolio Investors (FPI) stands for those investors who hold a short-term view of the company,
in contrast to Foreign Direct Investors (FDI). FPIs generally participate through the stock markets and get
in and out of a particular stock at much faster frequencies. A short-term view is associated often with a
lower stake in companies.
• Portfolio Investment by any single investor or investor group cannot exceed 10% of the equity of an
Indian company, beyond which it will be treated as FDI. Hence option (a) is the correct answer.
• Foreign Portfolio Investors are permitted to invest freely in Government Securities with a minimum
residual maturity of one year.
• The new FPI Regime came into effect on 1st June 2014 is based on recommendations of a “Committee
on Rationalization of Investment Routes and Monitoring of Foreign Portfolio Investments” under
the chairmanship of Shri K. M. Chandrasekhar with a view to rationalize/harmonize various foreign
portfolio investment routes and to establish a unified, simple regulatory framework.

Q 41.A
• The balance of payments (BoP) records the transactions in goods, services and assets between residents of
a country with the rest of the world for a specified time period typically a year. There are two main
accounts in the BoP — the current account and the capital account. Hence, statement 1 is correct.
• The essence of international payments is that , a country that has a deficit in its current account must
finance it by selling assets or by borrowing abroad. Thus, any current account deficit must be financed by
a capital account surplus, that is, a net capital inflow.
• A country is said to be in a balance of payments equilibrium when the current account deficit is
financed entirely by international lending without any reserve movements. Hence, if a country is
having BoP equilibrium it's official foreign exchange reserves remain unchanged. Hence, statement
2 is not correct.
• Alternatively, the country could use its reserves of foreign exchange in order to balance any deficit in its
balance of payments. The reserve bank sells foreign exchange when there is a deficit. This is called an
official reserve sale. The decrease (increase) in official reserves is called the overall balance of payments
deficit (surplus).
17 www.visionias.in ©Vision IAS
Q 42.C
• There are many ways to categorize poverty. In one such way, people who are always poor and those who
are usually poor but who may sometimes have a little more money (for example casual workers) are
grouped together as the chronic poor.
• Another group is the churning poor who regularly move in and out of poverty (example: small
farmers and seasonal workers) and the occasionally poor who are rich most of the time but may
sometimes have a patch of bad luck. They are called the transient poor. And then, there are those who are
never poor and they are the non-poor.
• Hence, option (c) is the correct answer.

Q 43.C
• Monkeypox is a viral zoonotic disease caused by the monkeypox virus. It is part of the
Orthopoxvirus genus which includes variola virus (smallpox) and cowpox virus.
• The disease is called monkeypox because it was first identified in colonies of monkeys kept for research
in 1958. It was only later detected in humans in 1970.
• It is usually a self-limited disease and typically lasts 2 to 4 weeks. It may be severe in children, pregnant
women or persons with immune suppression due to other health conditions
o The incubation period is usually 6 to 13 days and can range from 5 to 21 days.
o Typical symptoms include fever, headache, muscle aches, backache, lack of energy, swollen lymph
nodes and a skin rash or lesions.
o Swelling of the lymph nodes is a distinctive feature of monkeypox compared to other diseases that
may initially appear similar (chickenpox, measles).
• Transmission and risk of infection:
o Anyone can get or pass on monkeypox.
o A person with monkeypox remains infectious while they have symptoms, normally for between
2 and 4 weeks.
o Monkeypox virus is transmitted from one person to another by close contact with lesions, body
fluids and contaminated materials such as bedding, clothing or eating utensils.
o Ulcers, lesions or sores in the mouth can also be infectious, meaning the virus can spread through
saliva.
o People who closely interact with someone who is infectious, including health workers, household
members and sexual partners are at greater risk of infection.
o Transmission can also occur via the placenta from mother to fetus (which can lead to congenital
monkeypox) or during close contact during and after birth.
o It can spread to people when they come into physical contact with an infected animal. Animal hosts
include rodents and primates. The risk of catching monkeypox from animals can be reduced by
avoiding unprotected contact with wild animals, especially those that are sick or dead (including their
meat and blood). Hence option (c) is the correct answer.

Q 44.B
• A Soil Health Card (SHC) is a printed report card issued to farmers once in three years indicating
the status of their soil in terms of 12 parameters. It is also accompanied by a piece of advice on the
various fertilizers and other soil amendments he is supposed to make.
• The existing Nitrogen - Phosphorus - Potassium (NPK) consumption ratio in the country is highly
skewed to the preferred ratio of 4:2:1. SHC allows field-specific detailed reports of soil fertility status
and ensures the balanced application of fertilizers.
• The promotion of an integrated nutrient system is expected to reduce the consumption of chemical
fertilizers by 20% thereby easing the fiscal strain on Government. The fertilizer sector accounts for a
significant percentage of the total subsidies and power consumption in the country.
• India imports large quantities of various fertilizers to meet the demand (eg: 25-30 percent of Urea demand
is met through imports). Thus, the soil test-based fertilizer usage will reduce import bills and will also
ensure higher yields per unit.
18 www.visionias.in ©Vision IAS
• Over a period of time, SHC can determine changes in soil health that are affected by land
management.
• Hence, option (b) is the correct answer.

Q 45.D
• Hybrid Annuity Model (HAM) has been introduced by the Government to revive PPP (Public-Private
Partnership) in highway construction in India.
• HAM is a mix of The Build Operate and Transfer (BOT) Annuity and Engineering, Procurement and
Construction (EPC) Models. As per the design, the government will contribute 40% of the project
cost in the first five years through annual payments (annuity). The remaining payment will be made on
the basis of the assets created and the performance of the developer. Hence, statement 1 is not correct.
• Unlike the BOT model, under HAM there is no toll right for the developer. Under HAM, Revenue
collection would be the responsibility of the National Highways Authority of India (NHAI). Hence,
statement 2 is not correct.
• The advantage of HAM is that it gives enough liquidity to the developer and the financial risk is
shared by the government. While the private partner continues to bear the construction and maintenance
risks as in the case of the BOT (toll) model, he is required only to partly bear the financing risk. The
government’s policy is that the HAM will be used in the case of stalled projects where other models are
not applicable.

Q 46.A
• The patent system in India is governed by the Patents Act, 1970 as amended by the Patents (Amendment)
Act, 2005 and the Patents Rules, 2003
• Under the Atc, a Patent is a statutory right for an invention granted for a limited period of time to
the patentee by the Government, in exchange of full disclosure of his invention for excluding others,
from making, using, selling, importing the patented product or process for producing that product for
those purposes without his consent. Hence, statement 1 is correct.
• The term of every patent granted is 20 years from the date of filing of application. However, for
applications filed under the national phase under Patent Cooperation Treaty (PCT), the term of patent will
be 20 years from the international filing date accorded under PCT. Hence, statement 2 is not correct.
• Patent protection is a territorial right and therefore it is effective only within the territory of India. There is
no concept of global patent.
• An invention relating either to a product or process that is new, involving inventive step and capable
of industrial application can be patented in India. Hence, statement 3 is not correct.

Q 47.A
• Recently, Elections for 57 Rajya Sabha seats across 15 states are being held.
• Members of the Rajya Sabha are elected through single transferable votes via an open ballot. Members of
a state’s Legislative Assembly vote in the Rajya Sabha elections in what is called proportional
representation with the single transferable vote (STV) system. Each MLA’s vote is counted only once.
• Open ballot voting applies in elections to the Council of States only. Every political party which has
MLAs can appoint an authorised agent to verify whom its members have voted for. Hence, statement 1 is
correct.
• Members of Legislative Assembly (MLAs) belonging to a political party shall show their ballot papers
(after marking their vote) to the authorised agent of that party only and not to the authorised agent of
other parties. As such, the same person cannot be appointed as the authorised agent of more than one
party. Hence, statement 2 is correct.
• Independent MLAs are required to insert the marked ballot paper in the ballot box without showing the
marked ballot to any agent. Hence, statement 3 is not correct.

Q 48.D
• Recent Context: RBI lens on neobanks amid rapid growth in customers.
• A neobank is a kind of digital bank without any branches. Rather than being physically present at a
specific location, neobanking is entirely online. Hence statement 1 is correct.
• Neobanks are financial institutions that give customers a cheaper alternative to traditional banks.
19 www.visionias.in ©Vision IAS
• They leverage technology and artificial intelligence to offer personalised services to customers while
minimising operating costs.
o They are a broad collection of financial service providers, who primarily target tech-savvy customers,
comes under the umbrella of neobanking. Basically, a neobank is a fintech firm that provides digital
and mobile-first services like payments, debit cards, money transfers, lending, and more.
o These neobanks also offer branded debit cards. Hence statement 3 is correct.
• Neobanks bridge the gap between the services that traditional banks offer and the evolving expectations of
new-age customers. They do this by providing pers
• In India, these firms don't have a bank licence of their own but rely on bank partners to offer licensed
services. Hence statement 2 is correct.
o That’s because the Reserve Bank of India (RBI) doesn’t allow banks to be 100% digital yet.
o The RBI remains unwavering in prioritising banks’ physical presence, and has spoken about the
need for digital banking service providers to have some physical presence as well.

Q 49.D
• A proportional tax is an income tax system that levies the same percentage tax to everyone
regardless of income. A proportional tax is the same for low, middle, and high-income taxpayers.
Proportional taxes are sometimes referred to as flat taxes. Proportional taxation is intended to create
greater equality between marginal tax rates and average tax rates paid. Hence statement 1 is not correct.
• Proportional taxes reduce the autonomous expenditure multiplier because taxes reduce the marginal
propensity to consume out of income. The proportional income tax acts as an automatic stabilizer – a
shock absorber because it makes disposable income, and thus consumer spending, less sensitive to
fluctuations in GDP as compared to progressive taxation. Hence statement 2 is not correct.
o Progressive tax is the one where the tax rate increases with the taxpayer’s income. An example for
progressive taxation is: 10% tax rate for income of Rs 2 lakh, 20% for Rs 5 lakh and 30% for Rs 10
lakh. Here, the tax liability or the absolute amount as well as the proportion of income to be paid as
tax increases with the income of the taxpayer.

Q 50.A
• In a socialist society, the government decides what goods are to be produced in accordance with the
needs of society. It is assumed that the government knows what is good for the people of the country and
so the desires of individual consumers are not given much importance. Hence, statement 1 is correct.
• The government decides how goods are to be produced and how they should be distributed. In principle,
distribution under socialism is supposed to be based on what people need and not on what they can afford
to purchase. Unlike under capitalism, for example, a socialist nation provides free health care to all its
citizens.
• Strictly, a socialist society has no private property since everything is owned by the state. In Cuba
and China, for example, most of the economic activities are governed by socialistic principles.
• In a capitalist society, the goods produced are distributed among people not on the basis of what people
need but on the basis of Purchasing Power—the ability to buy goods and services. That is, one has to have
the money in the pocket to buy it. Hence, statement 2 is not correct.

20 www.visionias.in ©Vision IAS


Q 51.D
• Recently, Industry Transition Dialogue was hosted by India and Sweden in Stockholm. The event
was organised as a part of India and Sweden’s joint initiative — Leadership for Industry Transition
(LeadIT).
• Leadership for Industry Transition (LeadIT) was launched by the governments of Sweden and India at
the UN Climate Action Summit in September 2019 and is supported by the World Economic
Forum. Hence, statement 1 is not correct.
• Its Secretariat is hosted by Stockholm Environment Institute. Hence, statement 2 is not correct.
• LeadIT is a voluntary initiative for promoting low-carbon transition especially in the hard-to-abate
sectors like Iron & Steel, Aluminium, Cement and Concrete, petrochemicals, fertilisers, bricks, heavy-
duty transport, etc. through active participation of private sector companies.

Q 52.B
• Reforms have not been able to benefit agriculture, where the growth rate has been decelerating. Since
1991, public investment in agriculture sector especially in infrastructure, which includes irrigation,
power, roads, market linkages and research and extension (which played a crucial role in the Green
Revolution), has fallen. Hence, statement 1 is not correct.

• Further, the partial removal of fertilizer subsidy has led to an increase in the cost of production, which has
severely affected the small and marginal farmers.
• This sector has been experiencing a number of policy changes such as a reduction in import duties on
agricultural products, low minimum support prices and lifting of quantitative restrictions on the imports of
agricultural products. These have adversely affected Indian farmers as they now have to face increased
international competition.
• Moreover, because of export-oriented policy strategies in agriculture, there has been a shift from
production for the domestic market towards production for the export market focusing on cash
crops in lieu of production of food grains. This puts pressure on the prices of food grains. Hence,
statement 2 is correct.
Q 53.C
• Monetized deficit is the monetary support the Reserve Bank of India (RBI) extends to the Centre as
part of the government's borrowing program. In other words, the term refers to the purchase of
government bonds by the central bank to finance the spending needs of the government. Also known as
the ‘net reserve bank credit to the government’, it is that part of the government deficit which is
financed solely by borrowing from the RBI. Hence statement 1 is correct.
• Since borrowings from the RBI can be both short-term and long-term, therefore, the monetized deficit is
the sum of the net issuance of short-term treasury bills, dated securities (that is, long-term
borrowing from the RBI), and rupee coins held exclusively by the RBI, net of Government’s
deposits with the RBI. Hence statement 2 is correct.

21 www.visionias.in ©Vision IAS


Q 54.C
• The RBI controls the money supply in the economy in various ways. The tools used by the Central
bank to control money supply can be quantitative or qualitative.
• Quantitative tools control the extent of the money supply by changing the cash reserve ratio (CRR),
or bank rate or open market operations. Hence options 1, 2, and 4 are correct.
• If the Central bank changes the reserve ratio, this would lead to changes in lending by the banks which, in
turn, would impact the deposits and hence, the money supply.
• Qualitative tools include persuasion by the Central bank in order to make commercial banks
discourage or encourage lending which is done through moral suasion, margin requirement, etc.
Moral suasion is a tool of qualitative control. Hence option 3 is not correct.

Q 55.A
• Different countries have different methods of determining their currency’s exchange rate. It can be
determined through Flexible Exchange Rate, Fixed Exchange Rate or Managed Floating Exchange Rate.
• Flexible Exchange Rate is determined by the market forces of demand and supply. It is also known
as the Floating Exchange Rate. This is in contrast to a fixed exchange rate, in which the government
entirely or predominantly determines the rate. Hence, statement 1 is correct.
• In a completely flexible system, the Central banks do not intervene in the foreign exchange market. India
follows a managed floating exchange rate system. RBI acts as a controller in the exchange rate
market. Hence, statement 2 is not correct.

Q 56.C
• The TRIPS Agreement, which came into effect on 1 January 1995, is to date the most comprehensive
multilateral agreement on intellectual property.
• All the WTO agreements (except for a couple of “plurilateral” agreements) apply to all WTO members.
The TRIPS Agreement is part of that package. Therefore it applies to all WTO members. Hence,
statement 1 is correct.
• The TRIPS Agreement is often described as one of the three “pillars” of the WTO, the other two being
trade in goods (the traditional domain of the GATT) and trade in services.
• The TRIPS Agreement says WTO member countries must comply with the substantive obligations of the
main conventions of WIPO — the Paris Convention on industrial property, and the Berne Convention on
copyright (in their most recent versions).
• The TRIPS Agreement requires members to comply with certain minimum standards for the protection of
intellectual property rights covered in it.
• Article 31 of the Agreement allows compulsory licensing and government use of a patent without
the authorization of its owner. But this can only be done under a number of conditions aimed at
protecting the legitimate interests of the right holder. Hence, statement 2 is correct.

22 www.visionias.in ©Vision IAS


• Article 27.3(b) allows members to exclude some types of plant and animal inventions from patenting in
their countries. More specifically, it allows them to exclude from patentability “plants and animals
other than micro-organisms, and essentially biological processes for the production of plants or animals
other than non-biological and microbiological processes”.

Q 57.A
• The balance of payments account, which a statement of all transactions made between a country and the
outside world, consists of two accounts — current and capital account. While the current account deals
mainly with import and export of goods and services, the capital account is made up of cross-border
movement of capital by way of investments and loans.
• Current account convertibility refers to the freedom to convert your rupees into other internationally
accepted currencies and vice versa without any restrictions whenever you make payments. Similarly,
capital account convertibility means the freedom to conduct investment transactions without any
constraints. Hence, Statement 1 is correct.
• Since 1994, Indian rupee has been made fully convertible in current account transactions. But, India
has come a long way in liberating the capital account transactions in the last three decades and currently
has partial capital account convertibility. Hence, statement 2 is not correct.

Q 58.D
• The Reserve Bank of India was established on April 1, 1935 in accordance with the provisions of the
Reserve Bank of India Act, 1934.
• The Central Office of the Reserve Bank was initially established in Kolkata but was permanently moved
to Mumbai in 1937. Though originally privately owned, since its nationalization in 1949, the Reserve
Bank is fully owned by the Government of India.
• Important functions of Central Bank in an economy:
• Monetary Authority:
o Formulates, implements and monitors the monetary policy.
o Objective: maintaining price stability while keeping in mind the objective of growth. Hence
option 2 is correct.
• Regulator and supervisor of the financial system:
o Prescribes broad parameters of banking operations within which the country's banking and financial
system functions.
o Objective: maintain public confidence in the system, protect depositors' interest and provide cost-
effective banking services to the public.
• Manager of Foreign Exchange
o Manages the Foreign Exchange Management Act, 1999. Hence option 3 is correct.
o Objective: to facilitate external trade and payment and promote orderly development and maintenance
of foreign exchange market in India.
• Issuer of currency:
o Issues, exchanges and destroys currency notes as well as puts into circulation coins minted by
Government of India. Hence option 1 is correct.
o Objective: to give the public adequate quantity of supplies of currency notes and coins and in good
quality.
• Developmental role:
o Performs a wide range of promotional functions to support national objectives.
• Regulator and Supervisor of Payment and Settlement Systems:
o Introduces and upgrades safe and efficient modes of payment systems in the country to meet the
requirements of the public at large.
o Objective: maintain public confidence in payment and settlement system
• Related Functions:
o Banker to the Government: performs merchant banking function for the central and the state
governments; also acts as their banker. Hence option 4 is correct.
o Banker to banks: maintains banking accounts of all scheduled banks.

23 www.visionias.in ©Vision IAS


Q 59.C
• Open Market Operations refers to buying and selling of bonds issued by the Government in the
open market. This purchase and sale is entrusted to the Central bank on behalf of the
Government. Hence statement 1 is correct.
• RBI purchases (or sells) government securities to the general public in a bid to increase (or decrease) the
stock of high-powered money (monetary base) in the economy. Hence statement 2 is correct.
o Suppose RBI purchases Rs 100 worth of government securities from the bond market. It will issue a
cheque of Rs 100 on itself to the seller of the bond. The seller will deposit the cheque in her bank,
which, in turn, will credit the seller’s account with a balance of Rs 100. The bank’s deposits go up by
Rs 100 which is a liability to the bank. However, its assets also go up by Rs 100 by the possession of
this cheque, which is a claim on RBI. The bank will deposit this cheque to RBI which, in turn, will
credit the bank’s account with RBI with Rs 100. The total liability of RBI, or, by definition, the
supply of high-powered money in the economy has gone up by Rs 100.
o If RBI wishes to reduce the supply of high-powered money it undertakes an open market sale of
government securities of its own holding in just the reverse fashion, thereby reducing the monetary
base.
• High-powered money: The total liability of the monetary authority of the country, RBI, is called the
monetary base or high-powered money. It consists of currency (notes and coins in circulation with the
public and vault cash of commercial banks) and deposits held by the Government of India and commercial
banks with RBI.

Q 60.B
• Inflation-Indexed Bond (IIB) is a bond issued by a Sovereign, which provides the investor with a
constant return irrespective of the level of inflation in the economy. The main objective of Inflation-
Indexed Bonds is to provide a hedge and to safeguard the investor against macroeconomic risks in
an economy.
• For understanding the concept of IIB, it has to be compared with the instrument of fixed deposits with the
bank. While a fixed deposit offers a fixed rate of interest for the investment for a given number of years, it
does not protect the investor from the erosion of the real value of the deposit due to inflation. IIB on the
other hand gives a constant minimum real return irrespective of the inflation level in the economy. Capital
increases with inflation, so actual interest is better than originally promised. In the case of deflation,
interest payments decrease with negative inflation. However, capital does not decline below the face
value, ie. The initial investment, in case of deflation.
• A predecessor of Inflation Indexed Bonds (IIBs) was Capital Indexed Bonds (CIBs) issued in 1997.
However, the CIBs issued in 1997 provided inflation protection only to principal and not to interest
payment. IIBs provide inflation protection to both principal and interest payments. Hence statement
1 is not correct.
• There are no special tax concessions for these bonds. IIBs are treated as government securities (G-Sec)
and therefore, would be eligible for short-sale and repo transactions and gets SLR status (i.e., they
are eligible to be kept as part of Statutory Liquidity Ratio requirements of banks). Hence statement
2 is correct.

Q 61.C
• National Disposable Income is an aggregate macroeconomic variable that is used in National
Income accounting. It is the Net National Product at market prices plus net current transfers from rest of
the world.
• National Disposable Income= Net National Product at market price + net other current transfers
from the rest of the world. (Current transfers from the rest of the world include items such as gifts,
aids etc.). Hence statement 1 is correct.
• The National Disposable Income gives an idea about the maximum amount of goods and services
the domestic economy has at its disposal. Hence statement 2 is correct.
o It is the maximum available income (earned and transfer incomes) from all sources that a nation can
spend on consumption and saving without disposing of its assets to finance its expenditure.
24 www.visionias.in ©Vision IAS
Q 62.A
• The government provides certain goods and services which cannot be provided by the market
mechanism i.e. by the exchange between individual consumers and producers known as Public
Goods.
• The benefits of public goods are available to all and are not only restricted to one particular
consumer. One person’s consumption of a good does not reduce the amount available for
consumption for others and so several people can enjoy the benefits, that is, the consumption of many
people is not ‘rivalrous’. Hence statement 1 is correct.
• Examples of public goods are education, infrastructure, lighthouses, flood control systems, knowledge,
fresh air, national security, official statistics, etc. The public good is different from the common good
in that common good, though non-excludable, tends to be semi-rivalrous in nature. Examples of
common goods would be timber, coal, etc. Public goods are useful for the population as a whole.
• There is, however, a difference between public provision goods and public production
goods. Public provision means those goods and services that are financed through the budget and
can be used without any direct payment. Public goods may be produced by the government or the
private sector. When goods are produced directly by the government it is called public production
goods. Hence statement 2 is not correct.

Q 63.B
• Both the forms of capital formation (Capital and Human) are outcomes of conscious investment decisions.
Decision regarding investment in physical capital is taken on the basis of one’s knowledge in this regard.
Human capital formation is partly a social process and partly a conscious decision of the possessor of the
human capital.
• The owner of physical capital, say a bus, need not be present in the place where it is used; whereas, a bus
driver, who possesses the knowledge and ability to drive the bus, should be present when the bus is used
for transportation of people and materials.
• Physical capital is tangible and can be easily sold in the market like any other commodity. Human capital
is intangible; it is endogenously built in the body and mind of its owner. Human capital is not sold
in the market; only the services of the human capital are sold and, hence, there arises the necessity of the
owner of the human capital to be present in the place of production. Physical capital is separable from its
owner, whereas, human capital is inseparable from its owner.
• The two forms of capital differ in terms of mobility across space. Physical capital is completely mobile
between countries except for some artificial trade restrictions. Human capital is not perfectly mobile
between countries as movement is restricted by nationality and culture. Therefore, the physical
capital formation can be built even through imports, whereas human capital formation is to be done
through conscious policy formulations in consonance with the nature of the society and economy and
expenditure by the state and the individuals.
• Both forms of capital depreciate with time but the nature of depreciation differs between the
two. Continuous use of machine leads to depreciation and change of technology makes a machine
obsolete. In the case of human capital, depreciation takes place with ageing but can be reduced, to a large
extent, through continuous investment in education, health, etc. Hence, option (b) is not correct.
• Nature of benefits flowing from human capital are different from that of physical capital. Human capital
benefits not only the owner but also the society in general. This is called external benefit.

Q 64.C
• The Sagarmala Programme was approved by the Union Cabinet in 2015 which aims at holistic port
infrastructure development along the 7,516-km long coastline through modernisation, mechanisation
and computerisation.
• The initiative aims at integrating three things - the development of ports, industrial clusters and
hinterland and efficient evacuation systems through road, rail, inland and coastal
waterways. Therefore, components of Sagarmala include:
o Modernisation of port infrastructure- transforming the existing ports to world class ports and
development of new ports.
o Efficient evacuation system by improving hinterland linkages through rail, road and water.
25 www.visionias.in ©Vision IAS
o Encouraging coastal economic development by promoting port based SEZs and ancillary industries.
• Deep Ocean Survey and Exploration is a component of Deep Ocean Mission (DOM) launced by
Ministry of Earth Sciences.
• Hence, option (c) is the correct answer.

Q 65.D
• Türkiye, erstwhile Turkey (Capital: Ankara):
o UN has changed the name of Turkey to Türkiye after the Turkish Government requested it to
do so.
o It is a large peninsula that bridges the continents of Europe Asia Dardanelles, the Sea of Marmara, and
the Bosporus Strait.
o Political Boundaries: Bulgaria and Greece in the Northwest; Georgia, Armenia, Azerbaijan and Iran to
the east; and Syria and Iraq to the South.
o Black Sea connects it with Russia, Ukraine and Romania.
o Geographic Features:
▪ Inland Water Bodies: Lake Van (Biggest Lake) with Euphrates, Kizilirmak and Sakarya as main
Rivers.
▪ Highest Peak: Mt Arara

• Sudan (Capital: Khartoum):


o As per UN, the tribal clashes in Sudan’s war-ravaged Darfur region killed around 100 people.
o The fighting grew out of a land dispute between Arab and African tribes in the town of Kulbus
in West Darfur province. Local Arab militias then attacked multiple villages in the area, forcing
thousands of people to flee.
o The Darfur conflict began in 2003 when ethnic Africans rebelled, accusing the Arab-dominated
government in the capital of Khartoum of discrimination. Al-Bashir's government was accused of
retaliating by arming local nomadic Arab tribes and unleashing militias known as the janjaweed on
civilians there — a charge it denies.
o Sudan is a Northeast African Country sharing borders with 7 countries (Egypt, Libya, Chad, Central
African Republic, South Sudan, Ethiopia and Eritrea).
o South Sudan was separated from it in 2011.
o Dominated by massive Desert Plains (e.g. Nubian Desert) and Darfur Plateau, the Red Sea gives it
maritime access.
o Geographical Features:
o Highest Peak: Deriba Caldera (Jabel marra Mountain).
o Main River: Nile River with Khartoum as the merging point of its tributaries White Nile and Blue
Nile.

26 www.visionias.in ©Vision IAS


• Zimbabwe (Capital: Harare):
o Inflation soars more than 130% in Zimbabwe.
o Political boundaries: Landlocked country in south-central Africa, Bordered by Botswana on west,
Zambia on north, Mozambique on east, and South Africa on south. Geographical features
o Lake Kariba is world’s largest reservoir by volume Longest river: Zambezi
o Highest Point: Mount Inyangani.
• Cambodia (Capital: Phnom Penh):
o China is set to build its second foreign outpost (First at Dibouti) at Ream Naval Base
of Cambodia on the Gulf of Thailand.
o Cambodia is a Southeast Asian country surrounded by Thailand. Laos and Vietnam.
o The Gulf of Thailand gives it maritime access.
o Angkor Wat, an ancient temple complex built by Suryavarman I, located in northern Cambodia is the
world's largest religious structure.
o Geographical Features:
▪ Largest Lake: Tonlé Sap lake, largest freshwater lake of Southeast Asia, belonging to the Mekong
River System.
▪ Highest Peak: Phnom Aural in the Cardamom mountains.

• Hence option (d) is the correct answer.

Q 66.C
• The Gold Standard: From around 1870 to the outbreak of the First World War in 1914, the prevailing
system was the gold standard which was the epitome of the fixed exchange rate system. All currencies
were defined in terms of gold; indeed some were actually made of gold. Each participant country
27 www.visionias.in ©Vision IAS
committed to guaranteeing the free convertibility of its currency into gold at a fixed price. Hence,
statement 1 is correct.
• During 1914-45, there was no maintained universal system but this period saw both a brief return to the
gold standard and a period of flexible exchange rates.
• The Bretton Woods Conference held in 1944 set up the International Monetary Fund (IMF) and the World
Bank and reestablished a system of fixed exchange rates. A two-tier system of convertibility was
established at the center of which was the dollar.
• Many countries currently have fixed exchange rates. The international system is now characterized by
multiple regimes. Most exchange rates change slightly on a day-to-day basis, and market forces generally
determine the basic trends. Even those advocating greater fixity in exchange rates generally propose
certain ranges within which governments should keep rates, rather than literally fix them. Also, there has
been a virtual elimination of the role of gold.
• India’s exchange rate policy has evolved in line with international and domestic developments. Post-
independence, in view of the prevailing Bretton Woods system, the Indian rupee was pegged to the
pound sterling due to its historic links with Britain. Hence, statement 2 is correct.
• With the breakdown of the Bretton Woods system, and also the declining share of the UK in India’s trade,
the rupee was delinked from the pound sterling in September 1975. During the period between 1975 to
1992, the exchange rate of the rupee was officially determined by the Reserve Bank within a nominal
band of plus or minus 5 percent of the weighted basket of currencies of India’s major trading partners

Q 67.D
• Small Finance Banks (SFB) registered as a public limited company under the Companies Act,
2013 intend to provide financial services to the unserved and unbanked region of the country. Hence
statement 1 is correct.
• They primarily undertake basic banking activities of acceptance of deposits and lending to un-served
and underserved sections including small business units, small and marginal farmers, micro and small
industries, and unorganized sector entities, but without any restriction in the area of operations, unlike
Regional Rural Banks or Local Area Banks. Hence statement 2 is correct.
• Eligibility for Setting up SFBs:
o Resident individuals/professionals with 10 years of experience in banking and finance.
o The companies and societies are owned and controlled by residents.
o Existing Non-Banking Finance Companies (NBFCs), Micro Finance Institutions (MFIs), Local Area
Banks (LABs), and payment banks that are owned and controlled by residents.
• SFBs are full-fledged banks in contrast to payments banks created around the same time. Hence, they are
subject to all prudential norms and regulations of RBI as applicable to existing commercial banks like
maintenance of Cash Reserve Ratio (CRR) and Statutory Liquidity Ratio (SLR). Hence statement 3 is
correct.

Q 68.D
• Recent Context: The Securities and Exchange Board of India (SEBI), has reportedly told the
Parliamentary Standing Committee on Finance that regulation of crypto assets would be difficult
given the nature of technology that sustains them.
• Crypto assets is usually used as an umbrella term to encompass cryptocurrencies (e.g., Bitcoin, Ether) as
also non-currency tokens such as utility tokens (which provide a certain utility within an ecosystem) and
non-fungible tokens (which help establish ownership of unique items), among others. The underlying
technology for crypto assets is the same - distributed ledgers that aren't controlled by any one entity.
• The subscribers or customers of crypto assets form another important part of the market. SEBI has
suggested that the Consumer Protection Act of 2019 be invoked to make sure their interests are
safeguarded. As part of customer protection, SEBI had also proposed various measures to the
Advertising Standards Council of India. Hence statement 1 is not correct.
• SEBI has also sought clarity on whether cryptocurrencies can be legally classified as securities. Right
now, they aren't. SEBI has also said that crypto assets are not part of the definition of what
constitutes as securities under the Securities Contracts (Regulation) Act of 1956, also known as
28 www.visionias.in ©Vision IAS
SCRA. It is to be noted that what constitutes as securities under the SCRA is what is used under the SEBI
Act. Hence statement 2 is not correct.
• It also suggested possible regulation of crypto trading platforms by the RBI under the FEMA as
crypto assets are available for trading in foreign jurisdiction as well and consumers abroad can
remit funds to India using such currency. Hence statement 3 is not correct.
• While the government has introduced a taxation scheme on virtual digital assets, there is still no clarity on
their regulation. The government has also under-scored that the taxation scheme does not make cryptos
legal. In December 2021, it was widely reported that the government was looking to get SEBI to regulate
crypto assets by bringing in legislation around that time. Though such a legislation - the Cryptocurrency
and Regulation of Official Digital Currency Bill, 2021 - did not materialise at that time, the talk that the
government wants to treat cryptocurrencies as digital assets, rather than as currencies, has not ebbed.

Q 69.B
• Recently, India has questioned its bottom ranking among 180 nations on the Environmental
Performance Index.
• The Environmental Performance Index (EPI) is an international ranking system of countries based on
their environmental health. It is a biennial index, first started in 2002 as the Environment Sustainability
Index by the World Economic Forum in collaboration with the Yale Center for Environmental Law
and Policy and Columbia University Center for International Earth Information Network.
• EPI 2022 uses 40 performance indicators to assess and rank 180 countries. The 40 indicators are under the
broad categories of climate change performance, environmental health, and ecosystem vitality. The 2022
EPI has included new parameters to its earlier assessments, with projections of progress towards net-zero
emissions in 2050, as well as new air quality indicators, and sustainable pesticide use.
• Hence, option (b) is the correct answer.

Q 70.D
• Money aggregates are a broad category that measures the money supply in an economy. The total stock of
money in circulation among the public at a particular point of time is called the money supply.
• The Reserve Bank of India, RBI publishes figures for four alternative measures of money supply viz.
o M1= CU (Currency with Public) + DD (Demand deposits with the banking system) + Other deposits
with RBI
o M2= M1 + Saving deposits with post office saving banks
o M3= M1 + Net time deposits of commercial banks
o M4= M3 + Total deposits with post office savings organizations (excluding National Savings
Certificates)
• M1 and M2 are known as narrow money, M3 and M4 are known as broad money. Hence, statement
1 is not correct.
• M1 is the most liquid and easiest for transactions whereas M4 is least liquid of all. Hence statement
2 is not correct.
• M3 is the most commonly used measure of the money supply. It is also known as aggregate monetary
resources.

Q 71.B
• Core Inflation is also known as underlying inflation, is a measure of inflation which excludes items that
face volatile price movement, notably food and energy. In other words, Core Inflation is nothing but
Headline Inflation minus inflation that is contributed by food and energy commodities. Hence statement
1 is not correct.
o Unlike core inflation, headline inflation takes into account changes in the price of food and
energy. Since food and energy prices are highly volatile, headline inflation may not give an accurate
picture of how an economy is behaving.
• Prices of other commodities do not fluctuate as regularly as food and fuel – as such increase in their prices
could be taken relatively to be much more of a permanent nature. The Reserve Bank of India (RBI) and
Central Banks around the World always keep an eye on the core inflation. Whenever core inflation rises,
29 www.visionias.in ©Vision IAS
Central Banks increase their key policy rates to suck excess liquidity from the market and vice
versa. It is, therefore, a preferred tool for framing long-term policy. Hence statement 2 is
correct.

Q 72.B
• The Agriculture Ministry recently unveiled India’s first Covid-19 vaccine for animals. Developed by the
Hisar-based National Research Centre on Equines, the vaccine, called Ancovax, can protect animals
against the Delta and Omicron variants of SARS-CoV-2. Hence statement 1 is correct.
• Ancovax can be used in dogs, lions, leopards, mice, and rabbits. The vaccine can protect animals in the
zoo. It can also prevent transmission from companion animals to the humans. Hence statement 2 is not
correct.
o The aim of the vaccine is to protect endangered animals such as lions and tigers. India reported at
least nine Covid infections in Asiatic lions in Chennai zoo last year, with one of the lioness likely to
have died of it.
o The animals develop similar symptoms to humans – cough, cold, fever, and lung lesions.
• It is an inactivated vaccine developed using an infectious part of the Delta variant. In addition, it uses
Alhydrogel as an adjuvant to boost the immune response. Hence statement 3 is correct.

Q 73.D
• Free Trade Agreements (FTAs) are arrangements between two or more countries or trading blocs
that primarily agree to reduce or eliminate customs tariffs and non-tariff barriers on substantial
trade between them.
o In FTAs, tariffs on items covering substantial bilateral trade are eliminated between the partner
countries; however, each maintains an individual tariff structure for non-members.
• The key difference between an FTA and a PTA is that while in a preferential trade arrangement
(PTA) there is a positive list of products on which duty is to be reduced; in an FTA there is a
negative list on which duty is not reduced or eliminated. Thus, compared to a PTA, FTAs are generally
more ambitious in coverage of tariff lines (products) on which duty is to be reduced. Hence, option (d) is
the correct answer.

Q 74.B
• Under the Constitution, of India, agricultural marketing is a state (provincial) subject. While intra-
state trades fall under the jurisdiction of state governments, inter-state trading comes under Central or
Federal Government
• Agricultural Produce Market Committee (APMC) is a statutory market committee constituted by a
State Government in respect of trade in certain notified agricultural or horticultural or livestock products,
under the Agricultural Produce Market Committee Act issued by that state government. Hence both
statements 1 and 2 are correct.
• Once a particular area is declared as a market area and falls under the jurisdiction of a Market Committee,
no person or agency is allowed to freely carry on wholesale marketing activities. APMC Acts provide
that the first sale in the notified agricultural commodities produced in the region such as cereals,
pulses, edible oilseed, etc. can be conducted only under the aegis of the APMC, through its licensed
commission agents, and subject to payment of various taxes and fee. APMCs charge a market fee from
buyers, and a licensing fee from the commissioning agents (they are not prohibited) who mediate
between buyers and farmers. Hence, statement 3 is not correct.
• Exporters, processors, and retail chain operators cannot procure directly from the farmers as the product
is required to be channelized through regulated markets and licensed traders.

Q 75.B
• Ricardian equivalence is an economic theory that says that financing government spending out of
current taxes or future taxes (and current deficits) will have equivalent effects on the overall
economy.
30 www.visionias.in ©Vision IAS
• This means that attempts to stimulate an economy by increasing debt-financed government spending will
not be effective because investors and consumers understand that the debt will eventually have to be paid
for in the form of future taxes. The theory argues that people will save based on their expectation of
increased future taxes to be levied in order to pay off the debt and that this will offset the increase in
aggregate demand from the increased government spending.
• It is called ‘equivalence’ because it argues that taxation and borrowing are equivalent means of
financing expenditure. When the government increases spending by borrowing today, which will be
repaid by taxes in the future, it will have the same impact on the economy as an increase in government
expenditure that is financed by a tax increase today. Hence option (b) is the correct answer.

Q 76.C
• Gross External Debt is defined as the outstanding amount of those actual current liabilities, that
require payment(s) of principal and/or interest by the debtor, in the future as per the terms laid out
in the contract between the debtor and the creditor and that are owed to non-residents by the
residents of the economy.
o The definition of gross external debt includes debt incurred by both the Government and the private
sector(s) of the economy but does not take into account contingent liabilities that are liabilities arising
in the event of specific occurrences covered by the debt contract viz. default by a debtor on the
principal and/or interest of a credit.
• Long Term External Debt is defined as debt with an ‘Original Maturity’ of more than one year
while Short Term External Debt is defined as debt repayments on-demand or with an ‘Original
Maturity’ of one year or less.
• Short Term External Debt is classified into (a) Trade Credits (of up to 6 months and above 6 months
and up to 1 year) (b) Foreign Institutional Investors’ (FII) Investment in Government Treasury Bills
and Corporate Securities (c) Investment in Treasury-bills by foreign Central Banks and
International Institutions etc. and (iv) External Debt liabilities of the Central Bank and Commercial
Banks. Hence options 1 and 2 are correct.
• Long-Term External debt is further classified into (a) Multilateral Debt (b) Bilateral Debt (c) ‘IMF’
signifying SDR allocations to India by the IMF (d) Export Credit (e) (External) Commercial
Borrowings (f) NRI Deposits and (g) Rupee Debt. Hence options 3 and 4 are not correct.

Q 77.D
• Recently, President Ram Nath Kovind inaugurated the Sant Kabir Academy and Research Centre
Swadesh Darshan Yojana and paid tribute to the Bhakti saint, Kabir at Maghar (Uttar Pradesh).
• The Bhakti movement, which began in the 7th century in South India, had begun to spread across north
India in the 14th and the 15th centuries. One school within the Bhakti movement was the Nirguni
tradition and Sant Kabir was a prominent member of it. In this tradition, God was understood to be a
universal and formless being. Hence, statement 1 is correct.
• Kabir’s compositions can be classified into three literary forms – dohas (short two liners), ramanas
(rhymed 4 liners), sung compositions of varying length, known as padas (verses) and sabdas (words).
Hence, statement 3 is correct.
• Kabir is in modern times portrayed as a figure that synthesized Islam and Hinduism. He did not only
target the rituals and practices of both Hinduism and Islam, but also dismissed the sacred authority of
their religious books, the Vedas and the Quran. Kabir did use the name Rama in his poems, but he
clarified that he was not referring to the avatar of Vishnu, but a formless and general Hindu name for the
divine. Author Manu S Pillai writes that he even combined Allah and Ram. Hence, statement 2 is
correct.

Q 78.B
• Treasury bills are short-term money market (not capital market ) debt instruments issued by the
Central government with a maturity of one year or less. At present T-Bills are issued in three tenors,
namely, 91 days, 182 days, and 364 days.
• Treasury bills are zero-coupon securities and pay no interest. They are issued at a discount and redeemed
at the face value at maturity.
• In India, the Central Government issues both, treasury bills and bonds or dated securities while the State
Governments issue only bonds or dated securities, which are called the State Development Loans

31 www.visionias.in ©Vision IAS


(SDLs). Therefore, Treasury bills are issued by the Government of India and there are no treasury
bills issued by the State Governments. Hence, option (b) is correct.
• G-Secs carry practically no risk of default and, hence, are called risk-free gilt-edged instruments. T-bills
are debt instruments and they act as a liability to the Indian government.
• RBI allowed foreign investors to invest in T-bills to bring foreign funds into the market. Currently,
short-term investments by a foreign portfolio investors (FPI) should not exceed 20 percent of the total
investment of that FPI in either central government securities (including treasury bills) or state
development loans or corporate bonds. Recently, The Reserve Bank of India raised the investment limit
for FPIs in government and corporate bonds. The short-term investment limit has now been increased
from 20 percent to 30 percent.

Q 79.D
• Stockholm+50 is an international meeting convened by the United Nations General Assembly
(UNGA) in Stockholm, Sweden. Hence statement 1 is correct.
• Theme- Stockholm+50: A healthy planet for prosperity of all-our responsibility, our opportunity. Hence
statement 3 is correct.
• In 1968, Sweden first proposed the idea of the Stockholm conference. Hence statement 2 is correct.
• It commemorates 50 years since the 1972 UN Conference on Human Environment, which made the
environment a pressing global issue for the first time.
• Key principles are assistance to developing countries, safeguarding wildlife and natural resources, control
pollution, assert human rights etc.
• Significance of Stockholm Conference:
o It pushed the boundaries for a UN system that relied on the concept of state sovereignty and
emphasised the importance of joint action for the common good.
o The declaration explicitly acknowledged states' sovereign right to exploit their own resources
pursuant to their own environmental policies.
o The Stockholm Conference also put global inequality in the spotlight.
o The increasing awareness of environmental challenges over the past 50 years has led to the spread of
national environmental agencies and the growth of global environmental law.

Q 80.C
• Infrastructure Investment Trusts (InvITs) are mutual fund like institutions that enable investments
into the infrastructure sector by pooling small sums of money from multitude of individual
investors for directly investing in infrastructure so as to return a portion of the income (after deducting
expenditures) to unit holders of InvITs, who pooled in the money. Hence, statement 1 is correct.
• InvITs can invest in infrastructure projects, either directly or through a special purpose vehicle (SPV). In
case of Public Private Partnership (PPP) projects, such investments can only be through SPV.
• InvITs are regulated by the securities market regulator in India- Securities and Exchange Board of
India (SEBI). SEBI notified SEBI (Infrastructure Investment Trusts) Regulations, 2014, providing for
registration and regulation of InvITs in India. The objective of InvIT is to facilitate investment into the
infrastructure sector in India. Hence, statement 2 is correct.
• InvITs are very much similar to the Real Estate investment Trusts (REITs) in structure and operations.
InvITs are modified REITs designed to suit the specific circumstances in India.
• InvITs, as an investment vehicle, may aid:
o providing wider and long-term re-finance for existing infrastructure projects.
o freeing up of current developer’s capital for reinvestment into new infrastructure projects.
o refinancing/takeout of existing high cost debt with long-term low-cost capital and help banks free
up/reduce loan exposure, and thereby help them create headroom for new funding requirements.
Q 81.C
• In the process of production, all machines and equipment used to produce other goods, are subject to some
wear and tear. In economic parlance, this loss of capital which every economy has to suffer is called as
Depreciation. A part of capital goods produced in the economy must be devoted to replace this wear and
tear. Hence statement 1 is correct.

32 www.visionias.in ©Vision IAS


• Otherwise, the productive capacity of a nation would be depleted. This replacement of the capital used is
Capital Consumption Allowance.
• In this scenario, the investment expenditure of the firms is made up of two parts. One part is to buy new
capital goods and machinery for production. It is called Net Investment because the production capacity of
the firms can be expanded.
• Another part is spent on replacing the used-up capital goods or the maintenance of existing capital goods.
The expenses incurred for this are called Depreciation Expenditure. The total investment by firms
comprising these two amounts is called Gross Investment.
• Gross Investment = Net Investment + Depreciation
• Or, Net Investment = Gross Investment – Depreciation.
• The Net investment increases the production capacity and output of a nation if it is positive. This can
easily be verified at the level of a single plant: the number of new machines installed in any given year
must be greater than the machines that have been used up during that year.
• The governments decide and announce the rates by which assets depreciate and a list is published, which
is used by the different sections of the economy to determine the real levels of depreciations in different
assets.
• It should be noted that depreciation does not take into account unexpected or sudden destructions
or disuse of capital as can happen with accidents, natural calamities, etc. Hence statement 2 is
correct.

Q 82.D
• Goods and Service Tax (GST) is the single comprehensive indirect tax, operational from 1 July
2017, on supply of goods and services, right from the manufacturer/ service provider to the
consumer. It is a destination-based consumption tax with facility of Input Tax Credit in the supply
chain. It is applicable throughout the country with one rate for one type of goods/service. It has
amalgamated a large number of Central and State taxes and cesses. It has replaced large number of taxes
on goods and services levied on production/ sale of goods or provision of service.
• It is levied at all stages right from manufacture up to final consumption with credit of taxes paid at
previous stages available as setoff. In a nutshell, only value addition will be taxed and burden of tax
is to be borne by the final consumer.
• The phrase "destination-based tax" implies that the tax would accrue to the taxing authority which
has jurisdiction over the place of consumption which is also termed as place of supply.
• At the Central level, the following taxes have been subsumed:
o Central Excise Duty
o Additional Excise Duty
o Service Tax
o Additional Customs Duty commonly known as Countervailing Duty
o Special Additional Duty of Customs.
• At the State level, the following taxes have been subsumed:
o State VAT/ Sales Tax
o Luxury Tax
o Octroi
o Entry Tax i.e, taxes on the entry of goods into a local area for consumption, use or sale therein. (other
than those in lieu of octroi)
o Purchase Tax
o Entertainment Tax which are not levied by the local bodies; i.e. panchayats, municipalities and
District councils of autonomous districts can impose taxes on entertainment and amusements.
o Taxes on general advertisements
o Taxes on lotteries, betting and gambling. Hence options 1, 2 and 4 are correct.
• GST does not subsume stamp duties. Hence option 3 is not correct.

Q 83.A
• Recently, four Holy Relics of Lord Buddha are being taken to Mongolia for an 11-day exposition to
coincide with Mongolian Budhha Purnima celebrations.
• Stupas erected over the bodily relics of Buddha (Saririka stupas) are the earliest surviving Buddhist
shrines. It is said that Ashoka (272–232 BC), being an ardent follower of Buddhism, opened up seven of
33 www.visionias.in ©Vision IAS
these eight stupas, and collected major portion of the relics for enshrinement within 84,000 stupas built by
him in an effort to popularise Buddhism as well as the cult of the stupas. Hence, statement 1 is correct.
• At the age of 80, according to Buddhist beliefs, Buddha attained salvation in Uttar Pradesh’s Kushinagar
district. The Mallas of Kushinagar cremated his body with ceremonies befitting a universal king. His
relics from the funeral pyre were collected and divided into eight shares to be distributed among the
Ajathsatrus of Magadha, the Licchavis of Vaishali, the Sakyas of Kapilavastu, Mallas of
Kushinagar, Bullies of Allakappa, the Mallas of Pava, the Koliyas of Ramagrama and a Brahmana
of Vethadipa. The purpose was erecting stupas over the sacred relics. Two more stupas came up — one
over the urn in which the relics had been collected and the other over the embers. Hence, statement 2 is
not correct.

Q 84.C
• Pradhan Mantri Suraksha Bima Yojna (PMSBY) is an insurance scheme covering accidental death
risk of Rs.2 Lakh for a premium of just Rs. 12 per year (i.e. Rs 1/month as premium). It will cover all the
savings bank account and post office account holders of the age group 18 to 70 years for accidental
disability or death. Hence, statement 1 is not correct and statement 2 is correct.
• Pradhan Mantri Jeevan Jyoti Bima Yojana (PMJJBY) is an insurance scheme for the age group of
18-50 years covering both natural and accidental death risk of Rs. 2 lakh for a premium of Rs. 330 per
year (less than Rs. 1/day). i.e. it will cover all the savings account holders of the age group 18-50
for death due to any cause.
• Under PMSBY, the risk coverage will be Rs. 2 lakh for accidental death and full disability and Rs. 1
lakh for partial disability.
• Public sector general insurance companies or other general insurance companies that are willing to offer
insurance coverage to individuals on similar terms would offer and administer this scheme. The scheme is
delivered through banks including regional rural banks as well as cooperative banks. Hence,
statement 3 is correct.
Q 85.D
• A Patent is a statutory right for an invention granted for a limited period of time to the patentee by the
Government, in exchange for full disclosure of his invention for excluding others, from making, using,
selling, or importing the patented product or process for producing that product for those purposes without
his consent.
• An invention may satisfy the condition of novelty, inventiveness, and usefulness but it may not qualify for
a patent under the following situations:
o An invention that is frivolous or which claims anything obviously contrary to well-established natural
laws.
o An invention the primary or intended use or commercial exploitation of which could be contrary to
public order or morality or which causes serious prejudice to human, animal or plant life or health or
to the environment.
o The mere discovery of scientific principle or the formulation of an abstract theory or discovery of any
living thing or non-living substance occurring in nature.
o A novel method of agriculture or horticulture - Only products and processes qualifying for novelty are
patentable).
o Any process for medicinal, surgical, curative, prophylactic (diagnostic, therapeutic) or other treatment
of human beings or any process for a similar treatment of animals to render them free of disease or to
increase their economic value or that of their products.
o Plants and animals in whole or any part thereof including seeds, varieties, and species and
essentially biological processes for production or propagation of plants and animals other than
microorganisms (this means plants and animals cannot be patented whereas microorganisms
can be patented). Hence, option (d) is the correct answer.
o Inventions relating to atomic energy.
o An invention which, in effect, is traditional knowledge or which is an aggregation or duplication of
known properties of a traditionally known component or components.
o A literary, dramatic, musical or artistic work or any other aesthetic creation whatsoever including
cinematographic works and television products. (comes under the Copyrights Act protection and
therefore is left out of the purview of the Patents Act.)
o A mere scheme or rule or method of performing a mental act or method of playing a game.
o A presentation of information.
o A mathematical or business method or a computer program per se or algorithms.

34 www.visionias.in ©Vision IAS


Q 86.C
• The term commercial bank refers to a financial institution that accepts deposits, offers checking account
services, makes various loans, and offers basic financial products like certificates of deposit (CDs) and
savings accounts to individuals and small businesses.
• Assets are things a firm owns or what a firm can claim from others. In case of a bank, apart from
buildings, furniture, etc., its assets are loans given to public.
• Another asset that a bank has is bank reserves. Reserves are deposits which commercial banks keep
with the Central bank (RBI) and Vault cash.
o Assets = Reserves (Vault cash + deposits with RBI) + Bank Credit (Loans + investments). Hence
options 1 and 3 are correct.
• Liabilities for any firm are its debts or what it owes to others. For a bank, the main liability is the deposits
that people keep with it.
o Liabilities = Deposits (Deposits of public with commercial banks)
• Therefore, deposits of the public with commercial banks form a part of the liability of the bank.
Hence options 2 is not correct.

Q 87.B
• Recently, the BrahMos supersonic cruise missile was first tested from a land-based launcher in
Chandipur.
• The Integrated Guided Missile Development Programme, conceived and led by Dr A P J Abdul Kalam
includes a range of missiles including Prithvi, Agni, Trishul, Akash and Nag. Hence, statement 1 is not
correct.
• An Inter-Governmental Agreement was signed with Russia in Moscow in 1998 by Dr Kalam, who headed
the Defence Research and Development Organisation (DRDO), and N V Mikhailov, Russia’s then Deputy
Defence Minister. This led to the formation of BrahMos Aerospace, a joint venture between DRDO and
NPO Mashinostroyenia (NPOM), the Indian side holding 50.5% and the Russians 49.5%.
• BrahMos is a two-stage missile with a solid propellant booster engine. Its first stage brings the missile
to supersonic speed and then gets separated. The liquid ramjet or the second stage then takes the missile
closer to three times the speed of sound in the cruise phase. The missile has a very low radar signature,
making it stealthy, and can achieve a variety of trajectories. Hence, statement 2 is correct.
• The BrahMos has three times the speed, 2.5 times flight range and higher range compared to subsonic
cruise missiles.
• It operates on ‘Fire and Forget Principle’, adopting varieties of flights on its way to the target. Its
destructive power is enhanced due to large kinetic energy on impact. Its cruising altitude could be up to 15
km and terminal altitude is as low as 10 meters. It carries a conventional warhead weighing 200 to 300
kgs. Hence, statement 3 is correct.

Q 88.B
• Recent Context: According to a census conducted by Chilika Development Authority (CDA) in
collaboration with The Fishing Cat Project (TFCP), the Chilika Lake, Asia’s largest brackish water
lagoon, has 176 fishing cats.
• About twice the size of a typical house cat, the fishing cat is a feline with a powerful build and stocky
legs. The fishing cat is an adept swimmer and enters water frequently to prey on fish as its name suggests.
It is known to even dive to catch fish.
• Wetlands are the favourite habitats of the fishing cat. In India, fishing cats are mainly found in
the mangrove forests of the Sundarbans, on the foothills of the Himalayas along the Ganga and
Brahmaputra river valleys and in the Western Ghats. Hence, statement 1 is not correct.
• West Bengal declared the fishing cat to be its state animal in 2012. Authorities in Chilika also named the
cat as the lake's ambassador in 2020. Hence, statement 2 is correct.
• The fishing cat is listed as Endangered on the IUCN Red List, which means that it faces a high threat
of extinction in the wild. The Convention on International Trade in Endangered Species (CITES) lists the
fishing cat on Appendix II part of Article IV of CITES, which governs international trade in this species.
In India, the fishing cat is included in Schedule I of the Indian Wildlife (Protection) Act, 1972 and thereby
protected from hunting. Hence, statement 3 is correct.
35 www.visionias.in ©Vision IAS
Q 89.B
• The Second five-year plan's driving force was P.C. Mahalanobis. He placed the greatest emphasis on
strengthening the economy's industrial basis. The Plan covers the period from April 1956 through
March 1961. The second plan was to put India on a path toward industrialization.
• In addition, the decision to develop the Indian economy on socialist lines led to the policy of the state
controlling the commanding heights of the economy, as the Second five-year plan put it. This meant
that the state would have complete control of those industries that were vital for the economy. The
policies of the private sector would have to be complementary to those of the public sector, with the
public sector leading the way.
• Hence, option (b) is the correct answer.

Q 90.B
• The Reserve Bank of India and Government of India signed the Monetary Policy Framework Agreement
on 20 February 2015. MPC was set up with the responsibility for price stability and inflation
targeting. Hence statement 1 is correct.
• The Central Government constitutes the MPC through a notification in the Official Gazette. Altogether,
the MPC includes six members, - the RBI Governor (Chairperson), the RBI Deputy Governor in charge
of monetary policy, one official nominated by the RBI Board and the remaining three members would
represent the Government of India. Hence statement 2 is not correct.
• These Government of India nominees are appointed by the Central Government based on the
recommendations of a search cum selection committee consisting of the cabinet secretary (Chairperson),
the RBI Governor, the secretary of the Department of Economic Affairs, Ministry of Finance, and three
experts in the field of economics or banking as nominated by the central government.
• The three central government nominees of the MPC appointed by the search cum selection committee will
hold office for a period of four years and will not be eligible for re-appointment.
• Monetary Policy Committee is defined in Section 2(iii)(cci) of the Reserve Bank of India Act, 1934 and is
constituted under Sub-section (1) of Section 45ZB of the same Act (Reserve Bank of India Act,
1934). Hence statement 3 is correct.

Q 91.D
• Champion Service Sectors refers to the 12 identified sectors (identified by the Ministry of
Commerce) where the Government wants to give focused attention to promoting their development
and realizing their potential.
• It is a Central Sector, umbrella scheme of the Department of Commerce for the period 2019-20 to
2023-24. A dedicated fund of Rs. 5000 crores have been proposed to be established to support initiatives
for sectoral Action Plans of the Champion Sectors.
• These include Information Technology & Information Technology enabled Services (IT&
ITeS), Tourism and Hospitality Services, Medical Value Travel, Transport, and Logistics Services,
Accounting and Finance Services, Audio Visual Services, Legal Services, Communication Services,
Construction, and Related Engineering Services, Environmental Services, Financial Services, and
Education Services.
• Hence, option (d) is the correct answer.
• The biotechnology sector and Pharmaceuticals and Medical Devices sector have been classified under
Champion manufacturing sectors and not services sectors.

Q 92.B
• Recently, Iraq's 'pearl of the south' Lake Sawa has run dry amid water crisis that experts say is induced
by climate change, including record low rainfall and back-to-back drought.
• The construction of dams on the Euphrates in Turkey led to a reduction in river discharge in Iraq and
scarcity of the groundwater, which feeds the lakes.This led to a severe environmental change by drying
Sawa Lake in southern Iraq, expanding desertification and adding a new dry salty alluvial lake bed that
will provide vast material of mud, silt, and salt.
• It was formed over limestone rock and studded with gypsum formations and has no inlet or outlet.
36 www.visionias.in ©Vision IAS
• Lake Sawa appears in some old Islamic texts. It is said the lake miraculously formed on the day the
Prophet Muhammad was born in 570 A.D.
• Hence, option (b) is the correct answer.

Q 93.B
• Enhanced Access and Service Excellence-EASE evolved for four annual editions from Financial Year
2019 to Financial Year 2022. It has also catalysed reforms in diverse areas in Public Sector Banks.
• EASE 5.0 ‘Common reforms agenda’ has been developed for Public Sector Banks. Hence option (b)
is the correct answer.
• EASENext:
o PSB Manthan, 2022 was organised in April 2022. This program paved the way for broader program
“EASENext”. The EASENext will comprise of two initiatives:
▪ EASE 5.0 (common PSB reforms agenda) and
▪ Bank specific strategic 3-year roadmap (on the basis of individual bank’s business priorities).
• EASE 5.0:
o Under EASE 5.0, PSBs will be investing in new-age capabilities. It will also deepen the ongoing
reforms for responding to evolving customer needs, technology environment and changing
competition.
o It will focus on digital customer experience, as well as integrated & inclusive banking. It will also
emphasise on supporting agriculture and small businesses.
o All PSBs will create a bank-specific 3-year strategic roadmap, which will entail strategic initiatives
beyond EASE 5.0.
o The initiatives will implemented be across diverse themes namely, business growth, risk, profitability,
operation, customer service, and capability building.

Q 94.C
• Recent context: For the first time the Indian Council of Medical Research (IMCR) has issued guidelines
specifically for type 1 diabetes, which is rarer than type 2.
• Type 1 diabetes is a condition where the pancreas completely stops producing insulin, the hormone
responsible for controlling the level of glucose in blood by increasing or decreasing absorption to the
liver, fat, and other cells of the body. This is unlike type 2 diabetes — which accounts for over 90% of
all diabetes cases in the country — where the body’s insulin production either goes down or the cells
become resistant to the insulin. Hence statements 1 and 2 are not correct.
• Type 1 diabetes is predominantly diagnosed in children and adolescents. Although the prevalence is less,
it is much more severe than type 2. Unlike type 2 diabetes where the body produces some insulin and
which can be managed using various pills, if a person with type 1 diabetes stops taking their insulin,
they die within weeks. Hence statement 4 is not correct.
o The exact cause of type 1 diabetes is unknown, but it is thought to be an auto-immune condition
where the body’s immune system destroys the islets cells on the pancreas that produce insulin.
o Genetic factors play a role in determining whether a person will get type-1 diabetes. The risk of the
disease in a child is 3% when the mother has it, 5% when the father has it, and 8% when a sibling has
it.
o The presence of certain genes is also strongly associated with the disease. For example, the prevalence
of genes called DR3-DQ2 and DR4-DQ8 is 30-40% in patients with type 1 diabetes as compared to
2.4% in the general population.
• The diabetic ketoacidosis is a serious condition where the body has a high concentration of ketones, a
molecule produced when the body isn’t able to absorb glucose for energy and starts breaking down fats
instead. Hence statement 3 is correct.

37 www.visionias.in ©Vision IAS


Q 95.B
• Personal income (PI) includes all income which is received by all the individuals in a year.
• It also includes transfer payments such as LPG subsidies. The welfare payments are received by
households, but they are not elements of national income because they are transfer payments. Similarly, in
national income accounting, some income is attributed to individuals, which they do not actually receive.
• First, it should be noted that out of National Income NI, which is earned by the firms and government
enterprises, a part of profit is not distributed among the factors of production. This is called Undistributed
Profits (UP). We have to deduct UP from NI to arrive at PI, since UP does not accrue to the
households.
• Similarly, Corporate Tax, which is imposed on the earnings made by the firms, will also have to be
deducted from the NI, since it does not accrue to the households. On the other hand, the households do
receive interest payments from private firms or the government on past loans advanced by them. And
households may have to pay interests to the firms and the government as well, in case they had borrowed
money from either. So, we have to deduct the net interests paid by the households to the firms and
government.
• The households receive transfer payments from the government and firms (pensions, scholarship, prizes,
for example) which have to be added to calculate the Personal Income of the households.
• Thus, Personal Income (PI) = NI – Undistributed profits – Net interest payments made by
households – Corporate tax + Transfer payments to the households from the government and firms.
• The Transfer payments to the households from the government and firms are added to the National
Income to arrive at the PI. Hence option 4 is not correct.

Q 96.D
• Current Account is the record of trade in goods and services and transfer payments. Trade in goods
includes exports and imports of goods. Trade in services includes factor income and non-factor income
transactions. Transfer payments are the receipts which the residents of a country get for‘free’, without
having to provide any goods or services in return. They consist of gifts, remittances and grants. They
could be given by the government or by private citizens living abroad.

38 www.visionias.in ©Vision IAS


• Capital Account records all international transactions of assets. An asset is any one of the forms in which
wealth can be held, for example: money, stocks, bonds, Government debt, etc. Purchase of assets is a
debit item on the capital account. These items are Foreign Direct Investments (FDIs), Foreign Institutional
Investments (FIIs), external borrowings and assistance.

• Hence, option (d) is correct.

Q 97.D
• The current account is in balance when receipts on the current account are equal to the payments on the
current account. A surplus current account means that the nation is a lender to other countries and a
deficit current account means that the nation is a borrower from other countries. Hence, statement 1 is
not correct.
• Balance on Current Account has two components:
o Balance of Trade or Trade Balance
o Balance on Invisibles
• Balance of Trade (BOT) is the difference between the value of exports and the value of imports of goods
of a country in a given period of time. Export of goods is entered as a credit item in BOT, whereas import
of goods is entered as a debit item in BOT. It is also known as Trade Balance.
o BOT is said to be in balance when exports of goods are equal to the imports of goods. Surplus BOT or
Trade surplus will arise if the country exports more goods than what it imports. Whereas, Deficit BOT
or Trade deficit will arise if a country imports more goods than what it exports.
• Net Invisibles is the difference between the value of exports and the value of imports of Invisibles of a
country in a given period of time. Invisibles include services, transfers, and flows of income that take
place between different countries.
39 www.visionias.in ©Vision IAS
• A surplus in the current account of a country does not necessarily imply a surplus balance of trade. Even
when there is a deficit in the balance of trade, the surplus in the balance of Invisibles can result in a
current account surplus. Hence, statement 2 is not correct.

Q 98.C
• Bank rate is the rate charged by the central bank for lending funds to commercial banks. Hence option (a)
is not correct.
• Repo rate is the rate at which the central bank of a country (Reserve Bank of India in case of India) lends
money to commercial banks in the event of any shortfall of funds. Repo rate is used by monetary
authorities to control inflation. Hence option (b) is not correct.
• The rate of interest offered by the bank to deposit holders is called the ‘borrowing rate’ and the rate at
which banks lend out their reserves to investors is called the ‘lending rate’. The difference between the
two rates is called ‘spread’. Thus, it is the difference between the interest rates of borrowers and
depositors in a bank, which is often referred to as the profit appropriated by the bank. Hence
option (c) is the correct answer.
• Cash Reserve Ratio (CRR) is a specified minimum fraction of the total deposits of customers, which
commercial banks have to hold as reserves either in cash or as deposits with the central bank. CRR is set
according to the guidelines of the central bank of a country. Hence option (d) is not correct.

Q 99.D
• Factor cost is the cost of an item of goods or a service in terms of the various factors which have
played a part in its production or availability.
• The main factors of production are-Land. Labour, Capital and Entrepreneurship, which provides for Rent,
Wages, Interest and Profit respectively.
• Factor cost includes only payment to the factors of production. Hence, statement 1 is correct.
• It does not include any tax. Hence statement 2 is correct.

Q 100.B
• India began its New Economic Policy (NEP) in 1991, under the leadership of P. V. Narasimha Rao. It
refers to economic liberalization or relaxation in the import tariffs, deregulation of markets or opening the
markets for private and foreign players, and reduction of taxes to expand the economic wings of the
country.
• NEP mainly constitutes of Liberalisation, Privatisation, and Globalisation (LPG) reforms.
Liberalization as introduced to put an end to the restrictions and open up various sectors of the economy.
These reforms took shape in the following forms:
o Deregulation of Industrial Sector: Industrial licensing was abolished for almost all but certain
product categories - alcohol, cigarettes, hazardous chemicals industrial explosives, electronics,
aerospace and drugs, and pharmaceuticals; many goods produced by small scale industries
were deserved; in many industries, the market was allowed to determine the prices.
o Financial Sector Reforms: establishment of private sector banks, Indian as well as foreign; those
banks which fulfill certain conditions have been given the freedom to set up new branches without the
approval of the RBI and rationalize their existing branch networks.
o Tax Reforms: In order to encourage better compliance on the part of taxpayers many procedures
have been simplified and the rates also been substantially lowered.
o Foreign Exchange Reforms: as an immediate measure to resolve the balance of payments crisis,
the rupee was devalued (and not revalued) against foreign currencies.
o Trade and Investment Policy Reforms: they were aimed at: (i) dismantling of quantitative
restrictions on imports and exports (ii) reduction of tariff rates and (iii) removal of licensing
procedures for imports.
• Hence, option (b) is the correct answer.

You might also like